PhD Interview Questions and Answers (13 Questions + Answers)

practical psychology logo

Most PhD applications include an interview. This allows your university (and perhaps even your prospective supervisor) to discuss the PhD with you in more detail.

This article lists some of the most common PhD interview questions along with their answers. The goal is to help you prepare for a PhD interview and pass with flying colors.

1) How did you develop this proposal?

PhD interview questions

When responding to this question, demonstrate your thought process, research skills, and the evolution of your ideas. Let's choose the subject of "Renewable Energy Integration in Urban Planning" as an example.

Sample answer:

"My proposal on 'Renewable Energy Integration in Urban Planning' originated from my undergraduate thesis on sustainable cities. Intrigued by the potential of renewable energy in urban environments, I conducted a literature review to identify gaps in current research. This review highlighted a lack of comprehensive strategies for integrating renewable technologies at a city-wide level. I then consulted with experts in urban planning and renewable energy, which provided practical insights into the challenges and opportunities in this field. I designed a methodology that combines spatial analysis with energy modeling to explore optimal renewable energy integration in urban landscapes. This proposal represents an amalgamation of academic research, expert consultation, and innovative methodology development."

This answer is effective because it mentions a literature review demonstrates the ability to conduct thorough research and identify gaps in existing knowledge.

2) Why do you wish to pursue a PhD?

For this question, it's important to articulate your passion for the subject, your long-term career goals, and how the PhD program aligns with these aspects.

Let's choose the subject of "Artificial Intelligence in Healthcare" for this example.

"I am passionate about leveraging technology to improve healthcare outcomes, and pursuing a PhD in Artificial Intelligence in Healthcare aligns perfectly with this passion. During my Master's, I was fascinated by the potential of AI to revolutionize diagnostic processes and personalized medicine. I believe a PhD will provide me with the deep technical knowledge and research skills necessary to contribute significantly to this field. My goal is to develop AI systems that enhance medical diagnostics, ultimately improving patient care and treatment efficiency. This PhD program, known for its pioneering research in AI and strong healthcare collaborations, is the ideal environment for me to develop these innovations and achieve my career aspirations in healthcare technology."

This is a great answer because you clearly state that the PhD will provide the necessary skills and knowledge, indicating a clear understanding of the purpose of the program.

3) Why do you think you are the right candidate for this PhD program?

Discuss how your research interests align with the program's strengths and the faculty's expertise. Explain how the program's resources, courses, and research opportunities can help you achieve your academic and career goals.

"I am deeply passionate about environmental science, particularly in the area of sustainable urban development. This passion was ignited during my master's program in Environmental Studies at XYZ University, where I completed a thesis on urban green spaces and their impact on city microclimates. This research not only honed my skills in data analysis and GIS mapping but also highlighted the importance of interdisciplinary approaches to environmental issues. I am drawn to your PhD program at ABC University because of its innovative research on sustainable urban planning and the renowned work of Professor Jane Smith in this field. Her research aligns with my interest in integrating green infrastructure into urban planning to mitigate climate change effects. My perseverance, attention to detail, and ability to synthesize complex data make me an ideal candidate for this challenging program. Pursuing this PhD is integral to my goal of becoming an environmental consultant, where I plan to develop strategies for cities to reduce their environmental footprint."

This response is effective because it mentions particular aspects of your experience and the program, avoiding generic statements. It also outlines how the PhD fits into your career path.

4) What do you plan to do after you have completed your PhD?

Be specific about the type of career you aspire to, whether it's in academia, industry, research, etc. Explain how the PhD will equip you with the skills and knowledge for your chosen career path.

"After completing my PhD in Computational Neuroscience, I plan to pursue a career in academia as a university professor. My doctoral research on neural network modeling will provide a strong foundation for teaching and conducting further research in this area. I aim to develop innovative courses that bridge computer science and neuroscience, addressing the growing demand for interdisciplinary knowledge in these fields. Additionally, I intend to continue my research on applying machine learning techniques to understand brain function, which has potential implications for developing new treatments for neurological disorders. This academic pathway allows me to contribute significantly to both education and research in Computational Neuroscience."

This is a great answer because it connects the PhD research directly to future career plans.

It also articulates how your work can impact both academia and the broader field of Computational Neuroscience.

5) Why have you chosen this specific PhD program?

Mention specific aspects of the program that attracted you, such as the curriculum, research facilities, faculty expertise, or reputation.

Explain how the program aligns with your research interests or academic background.

"I chose the PhD program in Artificial Intelligence at MIT because of its cutting-edge research and interdisciplinary approach, which perfectly aligns with my academic background in computer science and my passion for machine learning. The program's emphasis on both theoretical foundations and practical applications in AI is particularly appealing. Additionally, the opportunity to work under the guidance of Professor [Name], whose work in [specific area, e.g., neural networks or AI ethics] has deeply influenced my own research interests, is a significant draw. This program is an ideal fit for me to further develop my skills and contribute to the field of AI, ultimately aiming for a career in AI research and development in the tech industry."

This answer connects your background and goals to the program's offerings.

Including a specific professor's name shows detailed knowledge about the program and faculty.

6) What impact would you like your PhD project to have?

When answering this question, convey both the academic significance and the potential real-world applications of your research. Let's choose a project focused on developing eco-friendly battery technologies for electric vehicles for this example.

"My PhD project aims to develop new eco-friendly battery technologies for electric vehicles (EVs), addressing both the environmental impact of battery production and the efficiency of energy storage. I hope my research will contribute to the academic field by advancing our understanding of sustainable materials for energy storage, potentially leading to publications and patents. Beyond academia, I envision this project significantly impacting the EV industry by providing a more sustainable and efficient battery alternative. This innovation could play a crucial role in reducing the carbon footprint of transportation and supporting global efforts towards a greener future. Ultimately, I aspire for my work to not only advance scientific knowledge but also drive real-world changes in how we approach energy sustainability in transportation."

This is an excellent answer because it connects the project to larger environmental goals and societal benefits. It also reflects a forward-thinking approach, demonstrating your understanding of the project's potential long-term implications.

7) What difficulties would you expect to encounter during this project?

It's important to demonstrate awareness of potential challenges and convey a proactive mindset toward problem-solving. Let's choose a project focused on the development of a novel AI-driven diagnostic tool for early detection of neurological diseases for this example.

"In developing an AI-driven diagnostic tool for early detection of neurological diseases, I anticipate several challenges. Firstly, the accuracy and reliability of the tool depend heavily on the quality and diversity of the data used for training the AI algorithms. Obtaining a comprehensive dataset that adequately represents the population can be difficult due to privacy concerns and data availability. Secondly, ensuring the AI model's interpretability to be clinically useful while maintaining high performance is another challenge, given the complexity of neurological diseases. To address these, I plan to collaborate with interdisciplinary teams, including data privacy experts and neurologists, to source and utilize data ethically and effectively. I also intend to continuously refine the AI model, focusing on both its predictive accuracy and clinical applicability. These challenges, while significant, present valuable opportunities for innovation and interdisciplinary collaboration."

This response is effective because it clearly outlines realistic challenges specific to the AI diagnostic tool project. It also presents a proactive approach to overcoming these challenges, showing problem-solving skills.

8) How will you fund this project?

When answering this question, show that you've thought about the financial aspects of your research and are aware of funding sources that are available and applicable to your project. 

"I have identified multiple funding sources to support my renewable energy research project at Stanford University. Firstly, I plan to apply for the DOE Office of Science Graduate Student Research (SCGSR) Program, which offers substantial support for projects focusing on sustainable energy. My proposal for this grant is already in progress, highlighting how my project aligns with the DOE's priorities in advancing clean energy technologies. Additionally, I'm exploring departmental fellowships at Stanford, particularly those aimed at renewable energy research. I am also keen on establishing industry partnerships, given the project's relevance to current energy challenges and the potential for collaborative funding and technological exchange. Last but not least, I will seek conference grants to present my research findings, which can lead to further academic collaborations and additional funding opportunities."

Notice how this answer mentions funding sources that align with the renewable energy focus of the project and the resources available at Stanford University.

9) Tell us about a time you experienced a setback

Focus on a situation relevant to your academic or research experience. Let's use a real-world example where a research experiment failed due to unexpected variables.

"During my Master’s thesis on the effects of soil composition on plant growth, I faced a major setback. My initial experiments, which involved growing plants in different soil types, failed to produce consistent results due to unanticipated environmental variations in the greenhouse. This was disheartening, especially as the deadline approached. However, I responded by reassessing my experimental setup. I consulted with my supervisor and decided to control more variables, such as humidity and temperature. I also refined my data collection methods to include more frequent soil and plant measurements. These adjustments led to more reliable results, and I successfully completed my thesis. This experience taught me the importance of adaptability in research and reinforced the value of meticulous experimental design."

This is a great answer because it shows how you’ve encountered and overcame a specific problem, demonstrating resilience and adaptability.

10) What are your strengths and weaknesses?

When answering this question, it's important to present a balanced view of yourself, showing self-awareness and a commitment to personal development. Choose strengths that are relevant to a PhD program and weaknesses that you're actively working to improve.

"One of my key strengths is my analytical thinking, which I demonstrated during my Master's project where I developed a novel algorithm for data analysis. This required me to not only understand complex theories but also apply them creatively to solve real-world problems. As for weaknesses, I sometimes struggle with overcommitment, taking on too many projects at once. This occasionally led to stress during my undergraduate studies. However, I am actively working on this by improving my time management skills and learning to prioritize tasks more effectively. I've started using project management tools and setting clear boundaries, which has already shown improvements in my workflow and stress levels."

This answer maintains a good balance between strengths and weaknesses. It also shows self-awareness, demonstrating a proactive approach to personal development.

11) Why have you chosen to study for a PhD at this university?

Mention specific aspects of the PhD program that attracted you. Explain how your research interests align with the work being done at the university.

"I am drawn to the PhD program in Astrophysics at Caltech due to its outstanding reputation in space research and the unparalleled resources available at the Owens Valley Radio Observatory. My research interest lies in the study of exoplanets, and Caltech's active projects in this area, such as the Zwicky Transient Facility, align perfectly with my academic goals. The opportunity to work under the guidance of Professor [Name], known for pioneering work in exoplanetary atmospheres, is particularly exciting. Additionally, Caltech's collaborative environment and emphasis on interdisciplinary research are conducive to my professional growth, providing a platform to engage with experts from various fields in astrophysics."

This response directly connects your research interests with ongoing projects and facilities at Caltech. It also shows you’ve done your research on faculty members and their work.

12) What can you bring to this research group?

Focus on your unique skills, experiences, and perspectives that will contribute to the research group's success. Let's choose the field of Biomedical Engineering at Johns Hopkins University for this example.

"As a prospective member of the Biomedical Engineering research group at Johns Hopkins University, I bring a unique combination of skills and experiences. My expertise in microfluidics, honed during my Master’s research, aligns well with the group’s focus on developing lab-on-a-chip devices for medical diagnostics. I have also co-authored two papers in this field, demonstrating my ability to contribute to high-impact research. Additionally, my experience in a start-up environment, where I worked on developing portable diagnostic tools, has equipped me with a practical understanding of translating research into applications. I thrive in collaborative settings, often bringing interdisciplinary insights that foster innovative problem-solving. I am excited about the prospect of contributing to the group’s ongoing projects and introducing fresh perspectives to advance our understanding and application of biomedical technology."

This response shows your relevant expertise, ability to work in a team, and the unique perspectives you can offer, positioning you as a valuable addition to the research group.

13) Do you have any questions for us?

Asking good questions demonstrates your motivation. It also shows that you’ve given some genuine consideration to the project and/or program you’re applying to.

Some questions you can ask the interviewer include:

  • What will the supervision arrangements be for the project?
  • What kind of training and skills sessions are offered as part of the PhD program?
  • How many other PhD students has this supervisor seen to completion?
  • Are there any major developments or partnerships planned for the department?
  • Are there likely to be any changes to the funding arrangements for the project?
  • What opportunities will I have for presenting my research?

Remember: you’re a good student, with lots of potential. You’re considering at least three years of hard work with this university. You need to know that you’ll get on with your supervisor, that your work will be appreciated and that there are good prospects for your project.

What to wear to a PhD interview

Wear formal attire for a PhD interview. Your best bet is to wear a suit. A navy blue suit is the best and most versatile option. No matter your gender, a suit is always very professional.

For men, wear a suit with a tie, dress shirt, and dress shoes. For women, wear a suit (pantsuit or skirt suit) with a blouse, or conservative dress, and closed-toe shoes.

When in doubt, it’s better to be slightly overdressed than underdressed. The goal is to make a professional impression and feel confident, without your attire distracting from the conversation.

What to expect from a PhD interview

At its core, a PhD interview will consist of questions that allow your potential supervisors to get to know you better and have an understanding of what you’d like to study, why you’ve chosen your field of study, and whether you’d be a good fit for the PhD program.

You should expect general questions to help the interviewer get a sense of your likes and dislikes, and your overall personality.

Next, expect questions about your personal motivations for studying a PhD. Your interviewer will also be interested in any relevant experience you have to qualify you to study this PhD.

In the next section, expect questions about your PhD project. You should be prepared to discuss your project idea in detail and demonstrate to the interviewer that you are the ideal candidate.

Last but not least, the interviewer will discuss your future ambitions and give you an opportunity to ask questions. Remember that this interview goes both ways.

It’s important to ask the interviewer relevant questions to show your engagement and the serious consideration you are giving their program.

You are preparing to spend several years of your life at this school. Think about what is important to you and what would make or break your decision to attend this university.

Prepare a list of questions ahead of the interview.

Understanding the interviewer’s point of view

During a PhD interview, interviewers are typically looking for a range of traits that indicate whether you are well-suited for the rigors of a doctoral program and a research career.

These traits include:

Intellectual Curiosity and Passion: A strong enthusiasm for the subject area and a desire to contribute to and expand knowledge in the field.

Research Skills and Experience: Demonstrable skills in conducting research, including designing experiments, collecting and analyzing data, and interpreting results. Prior research experience relevant to the PhD topic is often a plus.

Resilience and Perseverance: The capacity to handle setbacks and challenges, which are common in research, and to persist in the face of difficulties.

Collaboration and Teamwork: Although PhD research can be quite independent, the ability to work well with others, including advisors, faculty, and other students, is crucial.

Self-Motivation and Independence: The drive to work independently, manage one's own project, and stay motivated over the long term.

Fit with the Program: Alignment of the candidate’s research interests and goals with the strengths and focus of the PhD program and faculty.

These traits not only indicate your readiness for a PhD program but also your potential to contribute meaningfully to their field of study and succeed in a research-oriented career.

Related posts:

  • University Interview Questions (16 Questions + Answers)
  • Project Manager Interview Questions (14 Specific Questions + Answers)
  • Strength-Based Interview Questions (21 Questions + Answers)
  • Engineering Interview Questions (15 Questions + Answers)
  • Business Analyst Interview Questions (17 Questions + Answers)

Reference this article:

About The Author

Photo of author

PracticalPie.com is a participant in the Amazon Associates Program. As an Amazon Associate we earn from qualifying purchases.

Follow Us On:

Youtube Facebook Instagram X/Twitter

Psychology Resources

Developmental

Personality

Relationships

Psychologists

Serial Killers

Psychology Tests

Personality Quiz

Memory Test

Depression test

Type A/B Personality Test

© PracticalPsychology. All rights reserved

Privacy Policy | Terms of Use

Science underdog..JPG

  • Jan 27, 2020

PhD Interview Weekend: A guide for aspiring scientists

By Michael Whaby

First of all, if you’re here because you’ve been invited to interview for a PhD program: Congratulations! If otherwise, then I hope to offer some useful information to any considering or currently applying to graduate school. Last year, I applied to five and was invited to interview for four different PhD programs—three were biomedical science programs; one was a systems biology program. Each interview weekend had its own little twists, but they all essentially contained the same events on the schedule.

I’m going to try to guide you through some of the inevitably awkward dinners and other social interactions that will likely take place during the interview weekends. I am also going to help you see into the future to imagine what the interactions with interviewers might be like. From my experience, there are definitely some mainstream topics to expect talking about with each interviewer. By the end, you should be supplied with an arsenal of general information about what to expect for the PhD interview weekend.

( The bottom of the page contains a General PhD Interview Weekend Itinerary . This is just one example of how an interview weekend can be structured.)

Talking through the awkward

PhD interview weekend poster session.

During one interview weekend, after a morning introduction from the Dean, all of the faculty and students were gathered in one big room. After the faculty introduced themselves to the students, the students were to walk around the room and talk to those faculty members about their research. By far, this was the most awkward event that occurred during any of the interview weekends. The goal was to introduce yourself, talk quickly about research, then, finally, end the conversation and move on to the next faculty member. Kind of like researcher–student speed dating; more so just a really weird form of forced communication.

Traditionally, there are poster sessions held where students walk around and stop at posters that interest them—sometimes they even offer beer or wine to enjoy as you walk around and conversate with faculty members and current graduate students. In my opinion, the poster sessions are a much less awkward style of meeting faculty and talking about research. Also, if you’ve properly prepared for your application to that program, then you might even run into faculty whose research enticed you to apply there in the first place. Coming prepared and well-informed will make for easier, less awkward conversation.

Background information and curiosity will make for the best use of time during poster sessions. In fact, the background information will even help to boost curiosity should the topic at hand be of interest. Engage in conversation and ask questions about the research, the current students, past students, and the program itself. Remember, the interview is not just another assessment of your qualification-ness; you are also assessing if a particular program is the right fit . They want to make a good impression on you just as much as you want to for them. I denied offers from a few programs because I didn’t feel that they were good fits for me—even programs that were more “prestigious” than the one I finally chose. You ultimately should decide where you believe you will thrive.

Typical PhD interview weekend faculty dinner.

The PhD interview weekends always include at least one faculty dinner. There is usually a dinner with current graduate students as well, but this is a rather less tense and more casual vibe than the faculty dinners. From my experience, the faculty dinners come in two flavors: they can be at a faculty member’s house...or somewhere else. If the faculty dinner with the interviewing students is at a faculty member’s house, odds are that the hosting faculty member is ballin’. The house will likely be beautiful (to me, at least). Great food, as well as beer and wine, were served at these house dinners. I’ve also attended faculty dinners in banquet halls or in a lobby with a lot of space; good food, beer and wine are also served in these settings.

Something about having a beer (or wine) really helped ease some tension and made the conversations with faculty feel more casual—especially if they were drinking too. During one interview weekend, I actually drank a whole pitcher of beer at a campus bar before leaving for the faculty dinner, where more beer was served (NOT recommended, but I actually did get into that program).

Bottom line, be yourself . You are a part of the interview weekend, which means you obviously have something in common with everyone there: a passion for science. And while the dinners might not be the best opportunity to talk about science, its a great time to gauge whether these are people that you want to learn science with and from for the next 4-6 (or 7) years of your life.

The interviews

Typical PhD interview weekend interview.

Interviews. The most stressful part of the whole weekend. Doing well in the interviews requires some homework beforehand. You will be informed of which faculty members you will be interviewed by sometime before the interview weekend. They try to match you with faculty that you have expressed interest in meeting, but this often doesn't happen. Regardless, before going into the interview, you need to know yourself and your interviewer. That sounds ridiculous right? Of course you know yourself. And how the hell are you supposed to know someone you've never met before? (If you read this post , then you would know!) Let me clarify:

Know yourself

You are going to be asked many questions about yourself. To name a few: Why do you want to pursue a PhD? (Read about why I chose to here .) Why did you apply to this program? What are your research interests? Tell me about your previous research. Why did you suck so bad freshman year of college? Why did you drop this class? It says here in your personal statement ________________, could you explain that further?

Basically, know how to back up all of the information that they have of you from your application . Be able to explain why you sucked freshman year, and what you did to get to where you are now. Be able to show in your GPA when you started to see the impact of your hard work or new behavior. You should even have stories in your mind for each topic you touched on in your personal statement. Also, know your CV.

Lastly, be able to think and talk scientifically . If you did research, make sure you know that research. Talk to your former research mentor if you think that you need help better preparing. I put emphasis on this, because one of my interviews did not go so well because I didn't know some of the science behind the research that I did well enough. I had only one interviewer formulate a hypothetical scientific scenario, where I was challenged with a genetics-related molecular biology question. Just have your scientific thinking cap on.

Know the interviewer

I remember telling one of the professors that I had in the Biomedical Master's Program at the University of Pittsburgh that I got my first interview invitation to a PhD program. The first thing he told me was, "know your interviewers' research focuses. At least read some abstracts." He couldn't have been more right.

The overwhelming majority of time during the interviews was spent talking about the interviewers' research—whether you want to or not. This is particularly challenging when the interviewer does research that is of little interest to you, but, if any, this would be a time to act interested. If you can walk into that interview with questions about the interviewer's research, you are off to a great start.

Lastly, know the interviewer's position and role(s) in the program, and ask them questions about the program, or research, or anything that you want to know. You might find that some of them aren't prepared with talking points to cover 30-45 minutes, so you being able to contribute to the conversation and knowing a bit about who you are talking to will really help you and them.

So, that's all I have for now. There are many other great sources out there. I'd recommend checking out some blog-style articles written in Nature or Science magazines. They always have good stories told from people with a wide variety of experiences.

General PhD Interview Weekend Itinerary

Day 1 (Thursday or Friday) – usually a day to meet people and get comfortable

12:00pm Arrive somewhere on campus or at hotel and get settled in.

1:00-6:00pm Orientation, seminars and poster session.

6:00-8:00pm Dinner with faculty and/or graduate students, with beer and wine.

Day 2 (Friday or Saturday) – usually interview day

8:30am A light breakfast with other students to have something to throw up after interviews.

9:30am-10:00am First interview.

10:15am-10:45am Second interview.

11:00am-11:30am Third interview.

11:45am-12:15pm Fourth interview?

12:30pm-1:00pm Fifth interview??

1:15pm-2:30pm Lunch.

3:00pm-3:30pm Exit meeting.

4:00pm-6:30pm Relax back at the hotel, or wherever.

7:00pm-8:30pm Dinner with faculty and/or graduate students, with more beer and wine.

Day 3 (Saturday or Sunday) – usually a tour of the campus surroundings (“things to do here”)

9:30am-11:30am Tour of campus and the surrounding environment.

12:00pm-1:00pm Lunch.

1:00pm You made it. Go home.

Good luck everyone! Hope this helped.

#phd #gradschool #biology #medicine #research

  • Biomedical Career Insight

Recent Posts

The “Little” Guys: Making the most of smaller universities

To aspiring medical students, those just starting out, and those already on their way

Location and Lifestyle in Graduate School

Top 15 PhD Interview Questions and Answers

Top 15 PhD Interview Questions and Answers

In this article

HiPeople Platform - Candidate screening dashboard

Streamline hiring withour effortless screening.

Optimise your hiring process with HiPeople's AI assessments and reference checks.

Are you ready to conquer the pivotal challenge of PhD interviews? Your path to securing a coveted spot in a doctoral program hinges on your ability to navigate the often intricate world of PhD interview questions. In this guide, we'll delve deep into the types of questions you may encounter, equip you with effective strategies to tackle them, and provide insights to help you shine in your academic pursuits. Let's dive right in and decode the art of answering PhD interview questions with confidence and poise.

What is a PhD Interview?

A PhD interview is a critical step in the admissions process for doctoral programs. It serves as an opportunity for the admissions committee to get to know you better, assess your qualifications, and determine your suitability for the program. In a PhD interview, you'll typically engage in a conversation with faculty members or admissions representatives to discuss your academic background, research interests, motivations, and potential contributions to the program.

Importance of PhD Interviews

PhD interviews hold immense significance in the admissions process. Here's why they matter:

  • Assessment of Fit : Interviews help institutions assess whether you are a good fit for their program and research community. They want to ensure that your academic and research goals align with their offerings and objectives.
  • Personality and Interpersonal Skills : Interviews provide insight into your personality, communication skills, and how well you might collaborate with faculty and peers. Your interpersonal skills can be as important as your academic qualifications.
  • Clarification and Engagement : Interviews allow you to clarify any aspects of your application and engage in meaningful conversations about your research interests. This interaction can set you apart from other candidates.
  • Demonstration of Enthusiasm : By participating in an interview, you have the chance to demonstrate your genuine enthusiasm and passion for your field of study and the program itself.
  • Lasting Impression : A successful interview can leave a lasting positive impression on the admissions committee, potentially tipping the scales in your favor during the final decision-making process.

How to Prepare for PhD Interviews?

Effective preparation is key to a successful PhD interview. Here's what you need to consider:

  • Research the Program : Dive deep into the program's website, course offerings, and faculty profiles. Understand their research areas, recent publications, and ongoing projects. Be ready to discuss how your interests align with theirs.
  • Create a Strong CV and Personal Statement : Craft a compelling CV and personal statement that highlight your academic achievements, research experience, and motivations. Tailor these documents to reflect your passion for the field and your alignment with the program.
  • Practice Interview Responses : Practice answering common interview questions, both alone and with a friend or mentor. Focus on clarity, conciseness, and relevance in your responses.
  • Gather Questions to Ask the Interviewers : Prepare thoughtful questions to ask the interviewers. This shows your genuine interest and engagement. Examples include inquiring about ongoing research projects and the support structure for graduate students.
  • Dress and Present Professionally : Choose a professional and comfortable outfit for your interview. Dressing appropriately shows respect for the process, while confidence in your appearance can boost your self-assurance during the interview.

Remember that preparation is the foundation of success in PhD interviews. By understanding their significance and thoroughly preparing, you can approach your interviews with confidence and increase your chances of securing a spot in your desired program.

Types of PhD Interviews

PhD interviews come in various formats, each designed to assess different aspects of your candidacy and suitability for a doctoral program. Understanding these interview types will help you tailor your preparation effectively.

Structured Interviews

Structured interviews are highly organized and follow a predefined set of questions. They aim to objectively evaluate your qualifications and fit for the program. Here's what to expect:

  • Question Consistency : In structured interviews, all candidates face the same questions, ensuring fairness and comparability.
  • Focused Assessment : Interviewers assess your responses against specific criteria, such as academic background and research experience.
  • Formal Setting : These interviews often occur in a formal setting and may involve a panel of interviewers.

Unstructured Interviews

Unstructured interviews take a more conversational approach, allowing interviewers to explore your personality, motivations, and compatibility with their research group. Here's what makes them unique:

  • Open-Ended Questions : Interviewers ask open-ended questions, giving you the freedom to express yourself more spontaneously.
  • Personality Assessment : This format delves into your personal qualities, interpersonal skills, and enthusiasm for the field.
  • Informal Atmosphere : Unstructured interviews often create a more relaxed, informal atmosphere.

Panel Interviews

Panel interviews involve multiple interviewers assessing you simultaneously. While they can be intimidating, they offer diverse perspectives and insights into your candidacy:

  • Varied Perspectives : Different panel members may come from various academic backgrounds, providing a holistic assessment.
  • Multifaceted Questions : Be prepared for a range of questions, including those related to your research, motivations, and potential contributions.
  • Time Management : Managing your responses and interactions with multiple interviewers can be challenging but crucial.

Video Interviews

In the digital age, video interviews have become increasingly common, especially for international candidates. Excelling in a video interview requires additional considerations:

  • Technical Setup : Ensure your camera, microphone, and internet connection are reliable to avoid technical disruptions.
  • Virtual Presence : Convey your enthusiasm and professionalism through the screen by maintaining eye contact and using appropriate body language.
  • Background and Attire : Choose a clean, professional background and dress as you would for an in-person interview. Avoid distractions.

Understanding these interview types will help you tailor your preparation and approach to each specific format, increasing your chances of success in the PhD admissions process.

Research Experience and Background Interview Questions

1. tell me about your research experience and background in your field..

How to Answer: Start with a brief overview of your academic and research journey, highlighting key milestones, projects, and publications. Emphasize your expertise, the relevance of your work to the PhD program, and any unique contributions you've made.

Sample Answer: "I hold a Master's degree in [Your Field] and have been actively involved in research for the past five years. My work has primarily focused on [Specific Research Area], where I've conducted experiments on [Research Topic]. I've published several papers in reputable journals and presented my findings at international conferences."

What to Look For: Look for candidates who can effectively communicate their research experience, showcasing their passion for the field and the impact of their work. Assess the alignment between their research background and the program's objectives.

2. Can you discuss a specific research project you've worked on in detail?

How to Answer: Choose a significant research project and provide a structured overview, discussing the objectives, methodologies, key findings, and your role in the project. Be concise and clear in explaining the project's relevance.

Sample Answer: "Certainly, one of my most impactful research projects was centered around [Project Title]. The primary aim was to [Project Objective]. I employed [Research Methodology] and collaborated closely with [Team Members]. Our findings, which indicated [Key Finding], have the potential to [Potential Impact]."

What to Look For: Evaluate the candidate's ability to articulate complex research projects coherently, emphasizing their contribution, teamwork, and understanding of the research's implications.

Motivation and Goals Interview Questions

3. why do you want to pursue a phd in [your field].

How to Answer: Share your genuine passion for your field, explaining how a PhD aligns with your long-term goals and emphasizing your commitment to contributing to knowledge.

Sample Answer: "I am deeply passionate about [Your Field] because of its potential to address pressing global issues, such as [Relevant Issue]. Pursuing a PhD is the next logical step for me to delve deeper into these challenges, conduct innovative research, and ultimately make a meaningful impact."

What to Look For: Seek candidates who demonstrate a clear and authentic passion for their field and a well-thought-out rationale for pursuing a PhD.

4. What are your specific research interests and potential research topics for your PhD?

How to Answer: Discuss your research interests, explaining how they align with the department's expertise. Mention potential research topics and why they are relevant.

Sample Answer: "My research interests revolve around [Specific Research Area], and I'm particularly intrigued by [Specific Aspect]. I see potential for investigating [Potential Research Topic] because it addresses [Research Gap], and it complements the ongoing work at your department, especially with [Professor's Name]'s research in [Related Area]."

What to Look For: Look for candidates with well-defined research interests that align with the program and an understanding of how their interests fit within the department's expertise.

Academic Preparedness Interview Questions

5. how have your academic achievements prepared you for a phd program.

How to Answer: Highlight your academic achievements, emphasizing relevant coursework, grades, honors, or awards. Discuss how these accomplishments have equipped you for the challenges of a PhD.

Sample Answer: "During my undergraduate and master's studies, I consistently excelled in courses related to [Relevant Subjects], earning top grades. I was also honored with [Academic Award] for my outstanding performance in [Specific Course], which I believe demonstrates my ability to excel in rigorous academic environments."

What to Look For: Evaluate candidates' academic records, looking for a history of strong performance and evidence of their ability to excel in the PhD program.

6. How do you plan to manage the demands of a PhD program, including coursework, research, and potential teaching responsibilities?

How to Answer: Outline your organizational and time-management skills, discussing strategies for balancing coursework, research, and other responsibilities. Mention any prior experience in multitasking or teaching.

Sample Answer: "I recognize that a PhD program can be demanding, but I've honed my time-management skills through [Relevant Experience]. I plan to create a detailed schedule that allocates dedicated time for coursework, research, and any potential teaching duties, ensuring that I maintain a healthy work-life balance."

What to Look For: Seek candidates who demonstrate a proactive approach to managing the academic demands of a PhD, emphasizing their organizational skills and adaptability.

Research Proposal Interview Questions

7. have you developed a research proposal for your phd if so, can you provide a brief overview.

How to Answer: Summarize your research proposal briefly, outlining the research question, objectives, methodology, and potential significance. Be concise and showcase your innovative thinking.

Sample Answer: "Yes, I've developed a preliminary research proposal focusing on [Research Question]. The main objectives are to [Research Objectives]. I plan to employ [Research Methodology] to investigate this, and if successful, this research could contribute by [Potential Impact]."

What to Look For: Evaluate the candidate's ability to conceptualize a research project, the clarity of their proposal, and the alignment with the program's research priorities.

8. How does your research proposal align with the department's research priorities and potential advisors?

How to Answer: Explain the alignment between your research proposal and the department's strengths and potential advisors. Mention specific faculty members you are interested in working with.

Sample Answer: "My research proposal aligns closely with the department's focus on [Department's Research Focus]. I am particularly interested in collaborating with [Professor's Name], as their work in [Professor's Research Area] directly relates to my research proposal, and I believe their expertise would be invaluable."

What to Look For: Look for candidates who have thoroughly researched the department's strengths and potential advisors and can clearly articulate the alignment between their proposal and the program's resources.

Problem-Solving and Critical Thinking Interview Questions

9. can you describe a complex problem you've encountered in your research or academic work and how you approached solving it.

How to Answer: Share a specific example of a challenging problem, outlining the steps you took to address it. Emphasize your problem-solving skills and critical thinking.

Sample Answer: "In one of my research projects, we encountered a complex issue with [Problem Description]. To tackle this, I initiated a collaborative discussion with my team, conducted thorough literature reviews, and explored alternative approaches. Eventually, we devised a novel solution that not only resolved the problem but also contributed to the field."

What to Look For: Assess the candidate's ability to navigate complex challenges, their problem-solving strategies, and their commitment to finding innovative solutions.

10. How do you approach interdisciplinary collaboration and integrating different perspectives into your research?

How to Answer: Explain your approach to interdisciplinary collaboration, highlighting instances where you've successfully integrated diverse perspectives into your work. Emphasize the benefits of such collaboration.

Sample Answer: "I believe that interdisciplinary collaboration is essential for addressing complex issues. In my previous research on [Project Name], I collaborated with experts from [Related Discipline]. We integrated their insights, which enriched our project by [Outcome]. This experience has reinforced my commitment to embracing diverse perspectives."

What to Look For: Look for candidates who value interdisciplinary collaboration, demonstrate an ability to work effectively with experts from other fields, and can articulate the benefits of such collaborations.

Communication Skills Interview Questions

11. how do you communicate your research findings to both academic and non-academic audiences.

How to Answer: Discuss your communication skills, emphasizing your ability to convey complex ideas in a clear and accessible manner. Mention any presentations, publications, or outreach activities.

Sample Answer: "I believe in effective science communication. I've presented my research findings at conferences, where I tailored my presentations to engage both experts in my field and non-specialist audiences. Additionally, I've authored articles for [Publication], aiming to make my research accessible to a wider readership."

What to Look For: Evaluate candidates' communication skills, assessing their ability to adapt their message to different audiences and their commitment to disseminating their research beyond academia.

12. How would you handle a situation where your research findings faced skepticism or resistance from peers or advisors?

How to Answer: Describe a hypothetical scenario where your research findings were met with skepticism, and outline your approach to addressing this challenge diplomatically and professionally.

Sample Answer: "If my research findings were met with skepticism, I would first seek to understand the concerns of my peers or advisors. I'd be open to constructive feedback and engage in respectful discussions to address their doubts. If necessary, I'd reevaluate my methodology or findings and provide additional evidence to support my conclusions."

What to Look For: Look for candidates who demonstrate resilience, the ability to handle criticism professionally, and a commitment to constructive dialogue when faced with skepticism.

Professional Development Interview Questions

13. how do you stay updated with the latest developments and trends in your field.

How to Answer: Share your strategies for staying informed about the latest developments in your field, such as attending conferences, reading journals, or participating in online forums.

Sample Answer: "I stay updated by regularly attending conferences like [Conference Name], subscribing to relevant journals, and actively participating in online communities and forums where experts discuss emerging trends. This continuous learning helps me remain at the forefront of advancements in my field."

What to Look For: Assess candidates' commitment to professional development and their proactive approach to staying informed about the latest research and trends.

14. Can you discuss any leadership or mentoring experiences you've had in your academic or professional journey?

How to Answer: Highlight any leadership or mentoring roles you've undertaken, emphasizing your ability to lead and mentor others effectively.

Sample Answer: "I've had the privilege of serving as a mentor for junior researchers in my lab, guiding them through research processes and offering academic support. Additionally, I was elected as the president of [Student Organization], where I led a team of [Number] members in organizing [Event/Project], which greatly enhanced my leadership skills."

What to Look For: Seek candidates who have demonstrated leadership and mentoring abilities, showcasing their capacity to contribute positively to the academic community.

Conclusion and Fit with the Program Interview Questions

15. why do you believe you are an ideal fit for our phd program in [your field].

How to Answer: Summarize the key reasons why you believe you are a strong fit for the program, emphasizing your alignment with the department's goals, resources, and values.

Sample Answer: "I am enthusiastic about your PhD program because it aligns perfectly with my research interests in [Your Field]. The department's renowned faculty, well-equipped laboratories, and collaborative environment make it an ideal place for me to pursue my research goals. Furthermore, I appreciate the department's commitment to fostering interdisciplinary collaborations, which mirrors my approach to research."

What to Look For: Evaluate candidates' understanding of the program's strengths and their ability to articulate why they are an excellent fit based on their research interests and values.

How to Prepare for a PhD Interview?

Preparing for PhD interviews is a critical step in ensuring your success in the admissions process. We will dive into the key aspects of thorough preparation that will help you leave a lasting impression on the interview panel.

Researching the Program and Faculty

Before you step into the interview room, it's essential to have a deep understanding of the program you're applying to and the faculty members who might become your mentors. Here's how to do it effectively:

  • Program Insights : Explore the program's website, course offerings, and any recent publications or research projects. Understand the program's strengths and what sets it apart from others.
  • Faculty Profiles : Dive into the profiles of faculty members in your area of interest. Familiarize yourself with their research, publications, and current projects. Identify potential advisors who align with your research goals.
  • Department Culture : Seek insights into the department's culture and values. This can help you tailor your responses to demonstrate your alignment with their objectives.

Creating a Strong CV and Personal Statement

Your CV and personal statement are your opportunities to showcase your academic achievements, research experience, and motivations. Crafting these documents effectively is crucial:

  • CV Highlights : Ensure your CV highlights relevant academic accomplishments, research contributions, and any publications or presentations. Use a clear, concise format.
  • Personal Statement Impact : Your personal statement should be a compelling narrative of your academic journey, research interests, and why you're passionate about pursuing a PhD. Tailor it to the program you're applying to.
  • Demonstrate Fit : Emphasize how your academic background and research experience align with the program's goals and faculty expertise. Show how you can contribute to the department's research endeavors.

Practicing Interview Responses

Practice makes perfect, and practicing interview responses is no exception. Here's how to hone your interview skills:

  • Mock Interviews : Conduct mock interviews with friends, mentors, or career advisors. Practice answering common questions while receiving constructive feedback.
  • Video Recordings : Record yourself answering interview questions. Review the recordings to assess your body language, clarity, and confidence.
  • Storyboard Responses : Create storyboards for potential questions. Outline your responses with key points and examples to ensure you address the interviewers' expectations.

Gathering Questions to Ask the Interviewers

Preparing thoughtful questions to ask the interviewers not only demonstrates your interest but also helps you gain valuable insights:

  • Research-Based Questions : Ask questions related to ongoing research projects, faculty collaborations, and opportunities for graduate students. For example, inquire about recent publications or upcoming research initiatives.
  • Program Support : Seek information on the support structure for graduate students, including funding, resources, and mentorship opportunities.
  • Departmental Vision : Ask about the department's vision for the future and how they envision contributing to your academic and research growth.

Dressing and Presenting Professionally

Your appearance and demeanor play a crucial role in creating a positive first impression. Here are some tips for presenting yourself professionally:

  • Appropriate Attire : Choose professional attire that is appropriate for the interview setting. Dressing well shows respect for the process and your interviewers.
  • Grooming : Pay attention to grooming and personal hygiene. Ensure you feel comfortable and confident in your chosen attire.
  • Body Language : Practice good body language, such as maintaining eye contact, offering a firm handshake, and sitting up straight. These non-verbal cues convey professionalism and confidence.

Thoroughly preparing for your PhD interviews is your opportunity to shine and showcase your commitment to academic excellence and research. Each aspect of preparation contributes to a successful interview experience, increasing your chances of securing a spot in your desired program.

During the PhD Interview

The moment you step into the interview room is when your preparation meets the real test. This section will guide you through the crucial aspects of handling yourself during the interview, leaving a lasting impression on the interview panel.

Arriving Early and Being Punctual

Punctuality is a mark of professionalism and respect. Arriving early not only demonstrates your commitment but also helps you settle in and overcome any last-minute nerves. Here's how to ensure you're punctual:

  • Plan Your Journey : Calculate the travel time and consider potential traffic or delays. Aim to arrive at least 15 minutes before your scheduled interview time.
  • Venue Familiarity : If the interview is in person, visit the interview venue beforehand, if possible, to familiarize yourself with the location. This can help reduce anxiety on the interview day.
  • Backup Plans : Have a backup plan in case of unexpected circumstances, such as traffic jams or technical issues for virtual interviews.

Building Rapport with Interviewers

Building a positive rapport with the interviewers can go a long way in making a favorable impression. Here are some strategies to establish a connection:

  • Engage Actively : Be an active listener during the interview. Respond thoughtfully to the interviewers' questions and show genuine interest in their responses.
  • Respectful Interaction : Address the interviewers by their titles or preferred names, and maintain a respectful tone throughout the conversation.
  • Ask Clarifying Questions : If you're unsure about a question or need clarification, don't hesitate to ask. This demonstrates your commitment to understanding and providing thoughtful responses.

Communicating Clearly and Confidently

Effective communication is essential during your PhD interview. Clear and confident communication helps convey your ideas and qualifications effectively:

  • Clarity is Key : Ensure your responses are clear and concise. Avoid jargon or overly technical language that might be unfamiliar to some interviewers.
  • Confidence and Poise : Maintain a confident tone while speaking, even when discussing complex topics. Confidence can inspire trust in your abilities.
  • Practice Active Listening : Show that you're engaged by nodding or providing verbal cues to indicate you're actively listening to the interviewers.

Addressing Tough Questions with Poise

Challenging questions are a part of most PhD interviews. How you handle them can make a significant difference. Here's how to address tough questions with poise:

  • Take a Breath : If faced with a difficult question, take a moment to collect your thoughts. Don't rush into an answer. Composure is more important than speed.
  • Structured Responses : Structure your responses logically, addressing the question's core and providing relevant examples or evidence to support your answer.
  • Be Honest : If you don't know the answer to a question, it's okay to admit it. Offer to explore the topic further or express your eagerness to learn.

Demonstrating Enthusiasm and Passion

Your enthusiasm for your field of study and the PhD program should shine through during the interview. Here's how to convey your passion effectively:

  • Speak About Your Research : When discussing your research interests, convey your excitement and the potential impact of your work on the field.
  • Share Relevant Experiences : Highlight experiences that have fueled your passion for the subject matter. Discuss the projects or coursework that have influenced your journey.
  • Show Gratitude : Express your gratitude for the opportunity to interview and your eagerness to contribute to the academic community.

The interview is your moment to not only showcase your qualifications but also your personality, professionalism, and enthusiasm. By mastering these aspects during the interview, you'll leave a memorable impression on the interview panel.

Post-PhD Interview Etiquette

Once the interview is over, your job isn't done yet. Proper post-interview etiquette is crucial in maintaining a positive impression and handling potential next steps. We'll delve into the essential aspects of post-interview conduct.

Sending Thank-You Notes or Emails

Sending a thank-you note or email after your PhD interview is a courteous gesture that can leave a lasting impression. Here's how to do it effectively:

  • Promptness : Send your thank-you note within 24-48 hours of the interview to express your appreciation while your interview is still fresh in the interviewers' minds.
  • Personalization : Customize each thank-you message for each interviewer, referencing specific points of discussion or insights gained during the interview.
  • Express Gratitude : Express your gratitude for the opportunity to interview and your enthusiasm for joining the program.

Reflecting on the Interview Experience

After the interview, take some time to reflect on the experience. Self-assessment can be a valuable tool for personal growth and future interviews:

  • Strengths and Weaknesses : Identify your strengths during the interview and areas where you can improve. Reflect on the questions that challenged you.
  • Learning Opportunities : Consider what you've learned from the interview process. How can you apply this knowledge to future interviews or your academic journey?
  • Feedback : If you receive feedback from the interview panel, use it constructively to enhance your interview skills for future opportunities.

Preparing for Possible Follow-Up Interviews

In some cases, you may be invited for a follow-up interview, especially if you're a strong candidate. Be prepared for the possibility of additional interviews:

  • Research Continuation : Be ready to discuss your research interests in more depth. Prepare to elaborate on your proposed projects or potential contributions to the program.
  • Personalized Questions : Expect more specific questions related to your application or areas of interest. Demonstrate your commitment and passion for the program.

Handling Rejections and Acceptances

The outcomes of your PhD applications can be emotionally charged, especially when it comes to rejections and acceptances.

Coping with Rejection

Receiving a rejection can be disheartening, but it's important to remember that it's not a reflection of your worth or potential. Here's how to cope:

  • Seek Feedback : If possible, request feedback from the admissions committee. Understanding the reasons for rejection can help you improve your future applications.
  • Stay Resilient : Maintain a positive outlook and resilience. Rejections are a natural part of the application process and can lead to growth and self-improvement.
  • Explore Alternatives : Consider alternative options, such as reapplying in the future or exploring similar programs that align with your goals.

Evaluating PhD Offers and Making Decisions

If you're fortunate enough to receive multiple offers, you'll face the pleasant yet challenging task of evaluating and choosing the right program for you. Here's how to navigate this decision-making process:

  • Comparative Analysis : Create a spreadsheet or chart to compare the pros and cons of each program, considering factors like funding, research opportunities, faculty expertise, location, and support services.
  • Visit the Campuses : If possible, visit the campuses or engage in virtual tours to get a feel for the environment and culture.
  • Consult Mentors and Advisors : Seek guidance from mentors, advisors, or professors who can provide insights and advice based on your academic and career goals.
  • Trust Your Instincts : Ultimately, trust your instincts and choose the program that aligns best with your aspirations and offers the support and resources you need.

Handling the aftermath of PhD interviews, including sending thank-you notes, reflecting on your experiences, and managing outcomes, is an integral part of the journey. By following these guidelines, you can leave a positive impression and make informed decisions regarding your academic future.

Facing PhD interview questions may seem daunting, but with careful preparation and a confident approach, you can shine during this critical stage of your academic journey. Remember to showcase your qualifications, passion for research, and enthusiasm for the program. Your ability to communicate effectively and handle questions with poise will set you on the path to success.

As you reflect on your interview experience and await the outcomes, maintain a positive attitude and be ready to adapt and learn from each opportunity. Whether you receive an acceptance or a rejection, remember that every step in this process is a valuable learning experience that can help shape your future endeavors. Stay resilient, stay motivated, and keep pursuing your passion for knowledge. Your PhD journey is just beginning.

You may also like

Top 15 Flask Interview Questions and Answers

Top 15 Flask Interview Questions and Answers

Top 15 PL/SQL Interview Questions and Answers

Top 15 PL/SQL Interview Questions and Answers

Top 15 CNA Interview Questions and Answers

Top 15 CNA Interview Questions and Answers

Unlock the next level of your recruiting workflows.

  • Graduate School

“Tell Me About Yourself” PhD Interview Question & Expert Answers

Tell Me About Yourself PhD Interview

The “tell me about yourself” PhD interview question seems like something you do not need to prepare for or think about. But ignoring the importance of this interview question imperils your candidacy, since admissions committees scrutinize this part of the interview as much as other elements of your application, like your grad school statement of purpose , research resume , or statement of intent . The “tell me about yourself” PhD interview question is among the most common and difficult graduate school interview questions, but the right prep will help you start the interview and set the tone for the rest of your conversation.

This article will detail the purpose behind this tough interview question, show you ways to build a response, and provide you with expert sample answers to inspire you.

>> Want us to help you get accepted? Schedule a free strategy call here . <<

Listen to the blog!

Article Contents 9 min read

What is the purpose of the “tell me about yourself” phd interview question.

“Tell me about yourself” is not a trick question. We promise. Graduate school interviews are designed to learn more about you, beyond the grades, test scores, and academic achievements listed on your graduate school resume . The point of the “tell me about yourself” PhD interview question is to get at the heart of what defines you as a person, student, instructor, and scholar. In short, this question is truly your chance to stand out.

But the “tell me about yourself” PhD interview question is also an exercise in conciseness and self-awareness. The way you answer matters as much as the content, and it reflects a lot about how you express yourself, how you see yourself, and what about your personality, intellect, and past experiences motivates you to pursue such an advanced degree.

Want to ace your grad school interview? Check out this video:

How to Prepare and Structure Your Answer

Areas to cover.

1. Background 

PhD admissions officers are interested in your background, so you should start your answer with personal details like your name, where you were born, etc., and then mention more specific details, like your school, the degree you just finished, and what you’re doing currently, academically or professionally. You can also talk about other things unique to you, like if you’ve changed cities or countries to pursue your degree, and whether you are a parent. 

2. Personal Story

You can use the introduction as a springboard to talk about the “inciting incident” of your story; the moment or event that made you realize you wanted to pursue art, engineering, or medicine. It is unique to you, obviously, but you should also be mindful of not spending too much time on describing the incident but rather, using it as a way to transition to the next section.

You can talk about the lead-up to the story, and the aftermath. Did your perspective change? Were you motivated to find answers to a problem or remedy some injustice? Talk about your motivations, feelings, and emotions in detail to make your story authentic and personable, while not devolving your story into parody or self-seriousness. 

I think my passion for the fine arts started in my childhood church. Every Sunday I would stare at the murals depicting scenes from the Bible, mostly because of how different they were from traditional interpretations of religious imagery. The artist who designed these murals chose a more modern style to depict them. He used faceless figures and elongated shapes for their bodies instead of typical line drawings or classical imagery.

I was puzzled. But, at the same time, I thought it was bold. I realized how art can be freeing, in both style and subject. I saw how an artist expresses themselves through their choices and how those choices reflect their ideas, worldview, and state of mind. It was these possibilities that got me sketching. But that phase lasted only a few years, mostly because I had no talent for drawing.

However, my interest in the fine arts never went away. I knew that even if I could never be an artist, I could still have a career in research, restoration, and exploration of art history. I entered an undergrad in Art History with an emphasis on ancient art, which I buttressed with a part-time job working in the Winters Sculpture Gallery at Downtown University. It was working in the gallery that let me see how people truly interact with art. I was touched by the time and contemplation people put into an image or sculpture, regardless of the style or subject, the way people in churches sit, in silence, surrounded by what they consider holy and beautiful.

Exploring themes of worship and idolatry during my undergrad made me recall my own early childhood experiences. It was then that I decided that I wanted to go further in examining the never-ending relationship between art and religion. I had my mind set on going to Europe – Florence or Turin specifically – and going into depth on the exchange between art and religion throughout the Renaissance up to the present. I even started learning Italian.

However, it was around this time when news broke of the discovery of thousands of remains at residential schools, which, made me question what I was doing in a significant way. The Catholic Church was largely responsible, along with many others, for these horrors. After learning about this terrible news, I could not reconcile the fact that I was about to devote my life to the study of religious art based in Catholicism, while the same Catholic church actively participated in cultural genocide. It was a wake-up call. I realized that too much time and effort has been put into classical and Renaissance art, and I didn’t want to be part of that tradition.

I began researching how Indigenous cultures in Canada represent, interpret, and express their spirituality in ways that are far removed from Judeo-Christian spiritual practices. While doing this research, I felt in myself a desire to right the wrongs of the past. Rather than reinforcing the supremacy of one tradition, I wanted to learn about another so that I could help in preserving and disseminating it. Many had fought for centuries to preserve and pass down the rich, cultural legacy of Indigenous spirituality, despite the unyielding forces opposed to it and I wanted to participate in understanding it as much as I could.

When I entered the Master of Fine Arts program at Waterloo University, I sought out Dr. Patrick Bouvier, who identifies as Metis and researches storytelling practices unique to Indigenous cultures. Dr. Bouvier was kind enough to help me define my research interests, given my art history background, and it was through him that I found out that the boundaries between Christian and native spirituality are fluid and that many Metis incorporate Christian practices into their ceremonies.

I also became aware that Indigenous spirituality, by its nature, escapes definition and contextualization. It is less about holy texts, churches, and congregations and more a way of life, a way of understanding your relationships with nature, people, and the past. Even the term, “Indigenous spirituality” is problematic, given the poor job it does of relaying the complex beliefs of First Nations peoples.

Through Dr. Bouvier’s mentorship and guidance, I became acquainted with this school’s Indigenous Studies program. One of the aspects of this graduate program that stood out for me was that it was the first of its kind in North America, created with the input and guidance of First Nations representatives. The immersive aspect also intrigued me, as the program embraces the central role played by the environment and hands-on learning in Indigenous culture. I am eager to experience these traditional knowledge concepts and bond with the Elders who make themselves available to students in the tradition of Indigenous pedagogy. The fact that there are almost no programs or fields that marry traditional art history studies with Indigenous culture means that there is a dire need for further exploration, which is what I hope to achieve when I complete my PhD.

“Tell Me About Yourself” PhD Interview Sample Answer #2

I recently graduated from the Autonomous University of Sinaloa with a master’s degree in Astronomy and Astrophysics, where I presented a thesis on the life of stars (their birth, existence, and death) that questioned formation theories and examined the role of the cold dark matter model in classifying new stars.

The stars and universe have always fascinated me, ever since I was a boy growing up in Mexico City. In the capital, you don’t see a lot of stars. It’s for the same reasons that people living in large cities cannot see more than a few stars at a time: light and air pollution. I could never imagine that one day I would see a night sky blanketed with stars – as many stars in the sky as there are grains of sand on a beach. But it happened one night.

I remember when I was ten – I remember very clearly how old I was because this experience shaped me forever – I left the city to go visit my relatives in Oaxaca, which is very far from the Distrito Federal. It was the first time I left the city, and it was a long journey. But during the last few hours of the trip, we drove through the Sierra Madre del Sur mountain range.

I was mesmerized. At such a high elevation, so far from the city, the sky lit up with stars. That beautiful sight sparked more than just awe; it made me ponder my relation to time as well. That’s why I remember how old I was because, in that moment, I said to myself, “I am ten years old now, but will I remember this when I am twenty, thirty.

The “tell me about yourself” PhD interview question is nothing to fear, provided you prepare. Once you have your answer, your preparation should also involve mock interviews so that you can become comfortable with the format, time your answer, and make adjustments. You can reach out to PhD admission consultant s to help you practice because they have the most experience with the “tell me about yourself” PhD interview.

If you feel that you need some graduate school application help to guide you in the interview, or any part of the application process, don’t hesitate. But following the steps in this article can also help you in crafting an authentic response. When you do have an answer prepared, try rehearsing a few key points so that you know how to move your story along. You should practice reading it out loud to be comfortable with the material, but don’t go through the effort of memorizing lines, as this will make you seem inauthentic.

Interviewers ask this question as an icebreaker to ease the pressure and put interviewees at ease. But it also serves the real purpose of getting a candidate to talk about themselves and how their experiences (both academic and non-academic) have shaped their academic interests. 

You can answer the “tell me about yourself” question by telling a personal story of how you ended up at this interview. You can also start by talking about your present if you’re a recent grad or about what you want to achieve. The key is to create an easy-to-follow narrative that showcases what prepared you for the program you’re applying to. 

You can practice beforehand and time yourself, keeping your answer to around the two-minute mark to avoid incoherence and rambling. 

Do not talk about personal stories unrelated to your academic pursuits. You can certainly incorporate hobbies, passion projects, or general interests outside of academia, but try to weave all these elements of your personality into a convincing portrait of you as a PhD candidate. Also, avoid mentioning anything you are uncomfortable talking about later in the interview. For example, if you are uncomfortable discussing your family background, do not bring it up in the ‘tell me about yourself’ answer because the admissions committee might ask follow-up questions about it. 

PhD and postdoc interview questions can broach a number of topics, from asking about your professional ambitions to how you would apply for funding. Be prepared for personal and field-related questions; they will make up the bulk of your interview. 

You can practice your answer in front of a trusted colleague, mentor, or grad school advisor to get worthwhile feedback. We strongly encourage the use of mock interviews to get the best results. Your advisor, whoever they are, can provide tips and constructive criticism on how to improve your answer or where it excels. 

The answer is supposed to come from you and your personal experiences, so you should brainstorm and prepare your answer personally. While advisors and consultants can help you shape it and improve the delivery, only you can tell your own story.  

Every graduate school and every program have their own program-specific requirements, but the average requirements include a specific GPA,, a specific GRE score, and supporting documentation like a statement of purpose and letters of recommendation . 

Want more free tips? Subscribe to our channels for more free and useful content!

Apple Podcasts

Like our blog? Write for us ! >>

Have a question ask our admissions experts below and we'll answer your questions, get started now.

Talk to one of our admissions experts

Our site uses cookies. By using our website, you agree with our cookie policy .

FREE Training Webinar:

How to make your grad school application stand out, (and avoid the top 5 mistakes that get most rejected).

Time Sensitive. Limited Spots Available:

We guarantee you'll get into grad school or you don't pay.

Swipe up to see a great offer!

phd interview schedule

phd interview schedule

Community Blog

Keep up-to-date on postgraduate related issues with our quick reads written by students, postdocs, professors and industry leaders.

Questions to Ask During Your PhD Interview

Zebastian D.

  • By Zebastian D.
  • August 22, 2020

PhD Interview Questions to Ask

As someone applying for PhD positions, you’ll no doubt be thinking of and preparing for the interview with your potential supervisor(s). You’re absolutely right to be doing this and planning your answers to some of the PhD interview questions that they’ll probably ask you; make sure you’ve read our guide on this to help you prepare.

Remember though that the PhD student-supervisor relationship works both ways; as much as the potential supervisor is interviewing you for a PhD position within their lab, you should also have the mindset that you’re also interviewing the professor for the role of supervisor, and be ready to ask questions! Ultimately the key thing you want to know after your PhD interview is you are both a good fit for each other.

With that in mind, I’ve prepared a common list of questions that you should consider asking to help you decide if the supervisor and the research lab is the right fit for you. You don’t need to ask all these questions but instead use this list as a guide for picking what feels most important to you.

I should also note that some of the answers to these questions can usually be found through a quick Google search of the potential supervisor or looking at their university profile. So do think about which questions in particular you want to bring up in person at the PhD interview.

Now on to the questions….

How many PhD students have you supervised previously, and did they all gain their PhDs?

You’re asking this to firstly work out how experienced the professor is at supervising students, based purely on the numbers previously supervised. The reason to ask the second question of how many students gained PhDs is to get an idea of the supervisor’s track record of successful supervision. The lower the percentage of students that went onto complete their PhD under his or her supervision (and not leave the program early ), the more alarm bells that should be ringing for you. This of course shouldn’t be your only data point in the decision-making process, and you should try and find out more about why those that left their PhD program before completion, did so. Equally it’s also a possibility that some students have been successful in their PhD research in spite of a supervisor’s lack of support. A good way to get a truer sense of this is by speaking to the supervisor’s current and past students.

Whilst a supervisor’s successful track record can be reassuring, don’t be put off if they haven’t supervised many (or even any) students before, particularly if they’re still fairly new in the job. There are many other ways in which you can get a sense of the supervisor-student fit.

phd interview schedule

How many years does a PhD project usually take in your lab?

If you’re in the UK, a full time PhD should normally take you 3-4 years to complete , as reflected by most funding grants for PhD research being for this time frame. You want to know and have some reassurance that most students in this lab do finish within this time frame. Asking this question will also help you better understand the supervisor’s attitude towards completion time frames; is this someone that would have no issue with a student that’s been working on their PhD for 6 years or are they driven to help students complete ‘on time’.

What are the key milestones for progression that you expect from students?

This is a good follow on from the previous question. Some supervisors can be very ‘hands off’ and set no expectations on their students about deliverables and if this is the type of student-supervisor relationship you’re after (which some are), then perfect!

However, whilst a PhD project is an independent body of work, that doesn’t mean you have to do it in isolation. Having a supervisor that also acts as a mentor is important; a key aspect of this is to help keep you on track to complete your project ‘in time’, which is most effectively done using regular milestones.

The actual milestones will vary between supervisors but what you’re looking for in their response is some indication that they’ve actually thought about them. Examples of milestones may be the completion of the literature review within the first 6 weeks of starting, first experimental data captured by month 3 and first paper published by the time you end year 1.

phd interview schedule

How many other students do you supervise?

You ideally don’t want your supervisor to have too many (>5) other PhD students under his or her supervision at any one time, simply because of the dilution of their time that will naturally occur. Being part of an active research lab is a big advantage however, so you don’t necessarily want to be the only student under their supervision either. Remember that the professor may also be responsible for several Master’s and undergraduate students too so you just need to know what to realistically expect from him in terms of available time to meet with you regularly.

How often do you meet with your PhD students?

You don’t want a supervisor that’s too prescriptive in how you run your project, but you do want someone who you know you can rely on to meet with regularly. Some professors set weekly one-on-one or group research meetings that occur at the same time, day and venue; you know exactly what you’re getting here. Others tend to meet less frequently but still at regular intervals. A good balance would be to have catch up meetings every 2 weeks but it’s important to know upfront what the expectations are from both sides about how often to meet.

Finding a PhD has never been this easy – search for a PhD by keyword, location or academic area of interest.

How flexible is the direction of the PhD project?

At the PhD level of higher education, the supervisor is there to provide mentorship and guidance to help you avoid going in a completely wrong direction with your research. You should however expect to have the freedom to take your project in any direction you want to (within reason). This should be the case even if it means deviating from the original research questions that were proposed at the start; you and your supervisor should be in agreement before you start about how much flexibility there can be. Remember too that sometimes the project may have to stay closely aligned to the original plan if it’s required by the industry funder, so this decision may be out of the supervisor’s hands to some extent.

What funding is available for this project?

You should know by the time you come to interview if the project is to be self-funded or if there is specific funding associated with it. It may feel like an awkward question to ask but you need to be very clear on how much of a living stipend you should expect and if there is any additional funding for things such as conference travel, paying for journal publication fees or other bench fees; you don’t want any unpleasant surprises about finances when you’ve already started the PhD.

phd interview schedule

Do you expect there to be any changes in funding during the course of the project?

Specifically, you want to find out if there’s any risk that the funding associated with the project could be removed. Most often, funding bodies don’t transfer the entire monetary amount of the agreed funding up front in one lumpsum (which can be in excess of £75,000 for a 3-year studentship). Instead, payments are made in instalments and may be done so on the basis that certain milestones are met. For industry funded projects, for example, there may be a service work element (such as specialist analysis using university-based equipment) associated with the funding which will need to be delivered on time for the university to continue to receive money. In reality a complete loss of funding is unlikely to happen, but you should find out if this is at all a possibility of happening.

What is the source of the funding?

It’s important for you to understand how your project will be funded. As discussed in the previous question, the specific funder may place certain requirements on the university that need to be fulfilled to receive the funds. Don’t let this put you off applying or even impact your decision to take on the specific project, but it’s an important factor to be aware of.

Are there any opportunities to earn additional money as a PhD student?

Even if you will receive a stipend during the course of your research project, it comparatively won’t be a lot of money to live on. If you want it, the opportunity to earn extra money can make a big difference in managing your finances. This may in the form of one day/week working as a research technician or paid work preparing and delivering lectures to undergraduate students. It’s useful to know if these opportunities will exist to help you manage your expectations about your finances. Make sure you don’t let yourself feel obligated to take on this additional work however, even if it is paid; the priority will be ensuring your research progresses on schedule.

Will I have the opportunity and be expected to publish papers?

phd interview schedule

In the UK there is no requirement for you to have published any journal papers before you are awarded a PhD. Doing so however can go some way towards making your final viva that much easier, and also giving you a ‘head-start’ on your publication track record if you continue on into academia after your PhD.

You should get a sense of if your potential supervisor sees an opportunity for you to publish your research, if this is something that you want to do. Equally you should be aware of the supervisor’s expectations about publishing to avoid any potential conflict between your supervisor wanting you to publish work during your PhD and you wanting to wait until you’re in the post-doc stage before writing papers.

How many papers have previous PhD students published with you?

Knowing the answer to this should give you a good idea about the expectations and opportunities of publishing papers during your PhD. It’s certainly a positive sign to know that previous students have successfully published their research and is often a sign of a good supervisory system being in place.

How often does your research group present at academic conferences?

Having the opportunity to present your research at an academic conference is a key experience to have obtained during your time as a PhD student. Some supervisors actively encourage this and ensure that all funding applications include allocations for paying for conference related fees. Others are less convinced about the value of students going to conferences, particularly due to the additional expense of doing so, and may therefore not be as supportive of conference participation.

It’s useful to know what the norm is within the supervisor’s research group so that there are no surprises further down the line.

phd interview schedule

Is there funding support available for attending conferences?

Again, to be clear on expectations of funding and support for conference attendance, you should find out if there are funds specifically allocated for this purpose. If there aren’t, does the supervisor actively provide support to their students in applying for additional funding for this?

Are there courses and training sessions available for PhD students?

Find out if there are extra resources available to you should you want to use them. For example, do the supervisors students go on paper writing courses, or workshops on how to perform literature reviews? There are lots of new things that you’ll be doing during your PhD, especially at the beginning so it’s good to know that there’s external help available if and when you need it.

What are your past PhD graduates doing now?

This is an interesting one to find out from the supervisor. Are most of their graduates continuing their career development within academia or have many moved into industry work or even to a field completely different to their area of research? Ideally, you’d want this to align with your own career options. If most PhD holders have gone into industry whereas you want to pursue an academic career, you should try and find out why they ended up leaving academia. For example, did these graduates initially have aspirations of becoming professors themselves but were not able to do so or does your particular field normally open up more opportunities within industry?

What kind of support do you or the university provide for helping with jobs after?

In particular, what role does the supervisor play in helping their recent PhD graduates find their next job role? Do they have any connections within industry that they’d be able to help you network with? Or have any of their past PhD students stayed on in the lab as post-docs and are there resources in place for you to potentially do the same?

Will there be opportunities to teach undergraduate students?

The opportunity to give lectures to undergraduate students or lead tutorials with them can be a good way to earn some extra money during your PhD (note though that not all universities/departments formally pay PhD students to do this). Getting teaching experience is also important if you’re planning on continuing on down an academic career path at a university so it’s useful if you can gain some of this during your PhD.

Do you as a lab do any team activities together?

This will help you get a sense of the environment you’ll be working in for at least the next three years. Is this a lab with several PhD students and post-docs that make up an active ‘research family’? Does the team ever go out for lunch together or day trips away together to unwind? This can be a great way to build a sense of comradery in a research job that can often feel like you’re working alone in. Some supervisors actively encourage and get involved in nurturing a team environment whilst others are more hands-off, leaving the students to do their own thing.

phd interview schedule

What is the work environment like? Do students work in a shared office space?

Be clear on what your daily workspace will be like at the lab and university. Do all PhD students sit together in an open space or are there smaller office spaces for one or two students to work in? Some people prefer the buzz of an open space whilst others like the quiet of lone working. Either way, you should know what your work environment will look like for the next three years and plan accordingly (e.g. buy some noise cancelling headphones if you need some quite time in the open plan office).

I’ve given you a number of different questions to think about and ask your potential supervisor at your PhD interview. Not all of them may be relevant, or even appropriate to ask, so do think carefully about which ones you do want to bring up at the interview and which answers you could find out independently by either speaking to other students or looking online. Your research project and your experiences at the university will be so much more enjoyable if you can make sure you and your supervisor are a good fit for each other. The best way to do this is to ask questions!

New PhD Student

Starting your PhD can feel like a daunting, exciting and special time. They’ll be so much to think about – here are a few tips to help you get started.

Reference Manager

Reference management software solutions offer a powerful way for you to track and manage your academic references. Read our blog post to learn more about what they are and how to use them.

Science Investigatory Project

A science investigatory project is a science-based research project or study that is performed by school children in a classroom, exhibition or science fair.

Join thousands of other students and stay up to date with the latest PhD programmes, funding opportunities and advice.

phd interview schedule

Browse PhDs Now

phd interview schedule

Do you need to have published papers to do a PhD? The simple answer is no but it could benefit your application if you can.

What is a Research Instrument?

The term research instrument refers to any tool that you may use to collect, measure and analyse research data.

phd interview schedule

Guy is in the 3rd and final year of his PhD in Immunology and Microbiology at The University of Newcastle, Australia. His research primarily focuses on investigating roles of our immune system outside of the typical pathogen surveillance.

Chloe-Casey-Profile

Chloe is a 2nd year PhD student at Bournemouth University, researching the mental health of postgraduate researchers and is designing interventions that may improve their wellbeing.

Join Thousands of Students

Stanford University

Interviewing and Talking with Prospective Faculty

The graduate application process varies broadly and depends on your specific academic program.

The variations might include:

  • Admission decision based only on an online application
  • In the cases where you are admitted to work with a specific faculty member, you should contact faculty directly
  • A short list of applicants are invited for an on-campus interview trip
  • Applicants are admitted first, and then invited for an on-campus recruiting trip

Because of the range of possible application processes, it’s critical to learn about each program's requirements. For some graduate programs, you will need to directly contact faculty because a specific professor will decide whether to admit you as their own graduate student. In these circumstances, make sure to contact the faculty directly. (When contacting faculty for the first time, see the side box below for suggestions.)

It will be important in your decision-making process to determine if you envision working closely with that faculty member and if you have complementary working and communication styles. The faculty member will also want to assess your experiences and how you work by communicating with you. Thus you will want to sharpen your interview skills as you communicate with faculty.

Some graduate programs will invite you to conduct a phone interview or invite you to the university for a campus visit (common in the biosciences). The interview is your opportunity to more thoroughly demonstrate that you have what it takes to be in the graduate program. You will want to show your understanding and enthusiasm for the research that you have done. Some suggestions to prepare for the interview are provided below.

Bryen E Irving's portrait

My advice to the next generation of scholars is to never be afraid to ask for help. At times it may seem like brilliance is a singular, herculean effort, but a lot of great ideas have been shaped and molded from the minds of many. Whether it’s your advisors or peers, we’re all here to help. Never be afraid to acknowledge that you don’t fully understand something—collaboration is encouraged and celebrated.

— Bryen Irving, PhD candidate in Physics

1. Before the interview

  • For campus visits, it’s OK to ask what is expected of you and how to prepare (e.g., if your travel expenses will be covered, how to dress, if you should bring your CV, etc.).
  • Learn about the faculty and people you will be meeting or communicating with. Read about the research interests of the faculty, including abstracts or papers. Prepare at least 1-2 specific research questions for each interview.
  • Review the research that you conducted. If it was published or presented at a conference, reread the paper, abstract, or poster. Prepare a brief (1-2 minutes) oral summary of your past work. What was the research question? How did you address it? What did you specifically complete and achieve? What are some possible questions that faculty might ask?
  • Prepare detailed questions you have of the program you are considering. Generic questions (e.g., tell me about your program) indicate that you didn’t read the basics on their website, and so won’t leave a positive impression. Determine what’s important for you (specific research facilities, professional development activities, student groups, opportunities for collaborations, etc.) and research them online.
  • Determine and list the questions you have about the program, university, and location of where you are visiting. What are you hoping to see and learn?
  • Ask a peer or friend (e.g. a current grad student or postdoc who is familiar with grad school interviews) to help you sharpen your interview skills.
  • Come prepared to the mock interview in professional attire and with your materials (CV, papers, etc.).
  • If possible, video record your mock interview. Although many cringe at watching themselves, the video can be incredibly helpful in revealing blind spots.
  • If your interview will be conducted via Skype or another video platform, some helpful tips are provided in this YouTube video .
  • After the mock interview, ask your peer for honest and critical feedback. Listen actively without being defensive and allow your peer to speak openly, which will help you improve.

2. During the interview

  • For campus visits, dress appropriately (usually business casual, but be comfortable); be on time; organize your papers (e.g., résumés/CVs, slides or images, questions).
  • Even for phone/Skype interviews, dressing professionally will help you to mentally prepare for the interview.
  • Speak enthusiastically about your work. Highlight your research accomplishments and/or professional growth. If asked to speak about a weakness, phrase your answer in a forward-looking manner to demonstrate learning and growth, and awareness of your weaknesses.
  • You’ll want to sound positive and enthusiastic. But avoid excessive enthusiasm, which could be interpreted as naiveté or desperation. This is a tough balance to achieve, so practice with others.
  • Listen actively to your faculty interviewers as they talk about their research.
  • Ask questions, using your prepared lists. Take notes to remember comments and suggestions.

3. After the interview

  • Summarize your perceptions of the program, university, and environment. Make a table listing the pros and cons. List the people whom you met on your visit, and write a brief comment for each person to help you remember your interactions. Is it a good fit for you?
  • Email your interviewers and thank them for their time. Follow up if you promised to provide any materials. Even if you determine that you don’t wish to work with that faculty member, this isn’t the time to burn bridges, and you might bump into them in the future.
  • Reflect on your interview performance and make adjustments to strengthen your next interview.

contacting faculty for the first time

Your first contact with faculty is absolutely critical, because you don’t get second chances to make a first impression. You’ll need to craft an initial email message that will clearly communicate who you are, and why you’re reaching out to them specifically.

This can be very challenging because you’ll need to be clear and concise in a brief email message. Provided below is a sample email message with additional suggestions.

Don’t simply copy this message, but this example provides a template that can be used to customize your own initial email message.

Subject: Ecology PhD Program at Stanford

Dear Professor Peter Beak, I am currently a senior and McNair Scholar at UC Davis, and would greatly appreciate an opportunity to briefly speak with you about your research and the Ecology and Evolution PhD program at Stanford. I am seeking to pursue a PhD in Ecology, and my research advisor (Professor Emilio Laca) spoke highly about Stanford’s graduate program. I am particularly fascinated by studies on the influence of infectious diseases on population dynamics and community interactions. I have conducted similar research here at UC Davis, and also at Northwestern University using freshwater plankton, and won an oral presentation award at ABRMCS. These are further described in my attached CV. I carefully reviewed your website, and would greatly appreciate speaking with you on the phone (~15 min) to learn more about future directions of your research, particularly on developing mathematical models. I am available during these time slots. Please let me know if any of these work for you, and I’d be happy to offer more time slots if needed.

Sept 1, Wed 12-5 pm Sept 3, Fri 12-7 pm Sept 6, Mon 12-7 pm Sept 7, Tue 9-12 pm

Steve Lee McNair Scholar splee "at" ucdavis.edu (cell) 650-555-1234

  • Use a brief subject line. Avoid vague subjects (e.g., “question” or “request”).
  • Address them by their full name and professional title. Don’t use “Hi” or “Hey” or other informal greetings.
  • In the very first sentence, quickly summarize who you are and why you’re contacting them. If a person known to the professor suggested that you reach out to them, include that info also.
  • Explain why you’re contacting them specifically. Describe your highlights briefly. Attach CV and/or link to LinkedIn profile. Include other links as needed.
  • State your request; be specific. Make it easy for them to say yes to your request; provide ample times when you’re available.
  • Include your full name. If you’re in a graduate prep program, include info. Include your email and phone.

Stanford University

© Stanford University, Stanford, California 94305

How To Prepare for a PhD Interview

  • Katie Baker
  • September 26, 2023

phd interview schedule

So, you’ve written your personal statement, you’ve sent through your application and now you’ve been invited to a PhD interview, the first step is to congratulate yourself; only 30 – 50% of PhD applicants make it through to the initial interview stage. The invitation is a sure-fire sign the admissions team were impressed by your academic track record, essays, research proposals and personal statement, but you aren’t out of the woods and in the door just yet. 

PhD interviews are conducted by universities to discuss the PhD program or your research proposal with you in more detail. You can also expect to be asked questions about your academic background, skills, and career goals. By asking these questions, university admissions teams will assess whether you are capable enough to carry out doctoral research. With enough preparation, your PhD interview doesn’t need to be daunting. 

To help you prepare for your interview, this page will outline advice on how to prepare for a PhD interview and give you an insight into what typically happens at PhD interviews. However, it is worth bearing in mind that each university interviews its potential PhD candidates differently. 

Along with following the advice outlined here, you will also want to ensure you have a good idea of what to expect from your interview. For example, some university boards will ask you to do a short presentation; others will only ask you questions based on your research proposal and other application materials that spurred them to pick up the phone and invite you to elaborate on your application. 

Know What To Expect From a PhD Interview

Even though PhD interview questions can vary significantly from institution to institution, depending on the circumstances of your PhD application or be formulated specifically around your subject area, every interview serves the same purpose. 

So while it is difficult to give you an idea of a standard PhD interview format, the principle is the same; you will be discussing the finer details of your research proposal or your academic background if you are applying for a program with pre-determined aims and objectives. 

Formal interviews will put you in front of a postgraduate recruitment panel, potentially comprising admission tutors and PhD supervisors; on the other end of the spectrum, you could be asked to meet your potential supervisor for lunch or in a coffee shop on campus to discuss your research interests. Some universities even choose to include orientation activities in the initial interview process; this will give you the chance to explore the research facilities and meet staff members and peers. 

Whichever setting your interview takes place in, and whoever is present, remember that the focus will always be on your achievements, academic interests, and goals.

PhD Interview Presentations

If a PhD interview presentation is required, you will be informed by the university well in advance to allow you to prepare. Your prospective department will also outline their expectations for the presentation, including how long it should be, what needs to be covered, and how it should be delivered. 

Typically, PhD interview presentations should take no longer than 15 minutes to complete, be delivered via PowerPoint and cover your academic achievements and background, research methods and the impact of your research. However, for students interviewing for advertised positions, there may be a requirement to give a short presentation on a specified topic related to your field.

How Long Does a PhD Interview Take?

There is no one-size-fits-all answer for how long a PhD interview takes; the duration depends on the format, how quickly you provide the information that the postgraduate recruitment panel and several other factors.

With some interviews, you should block out an entire day in your calendar as the university may invite you to meet the recruitment panel in the morning, have lunch on campus, explore the department, and have your formal interview afterwards. With others, you can expect to meet your future supervisor and speak with them for an hour over coffee. Whichever interview format you are invited to, remember you’ll be doing the degree be prepared to make the most of it!

row of students sitting on a bench waiting next to eachother

PhD Interviews for Advertised Positions

Most STEM PhD programs come with pre-defined aims, usually part of a research program with broader research objectives; your doctoral research will contribute to the body of research. Furthermore, many advertised PhD projects will also have secure funding. With these kinds of positions, all applicants must prove via their interviews that they can carry out the research to a high standard and prove they deserve the secured funding or studentship.

For example, imagine a particular PhD degree involving the analysis of a particular protein. Being a talented and competent life scientist can get you the interview, but exhibiting your knowledge of the specific proteins and familiarity with the techniques and equipment you need to run your analytical research will seal you the position. You don’t need to be an expert; you need to prove your capacity to become an expert – given the chance.

With advertised positions, the interviews are typically more formal, and the main component will comprise a question-and-answer session where you will be in front of a qualified postgraduate recruitment panel involving three or more people, including project supervisors, postgrad admissions staff, lead investigators and funding reps.

The panel will focus on your research goals and interests and how they correlate to your academic background. You may also be expected to expand on sections of your application which need more clarification. 

It isn’t heard of for interviewers to ask you to give a specific presentation in addition to answering questions. However, this presentation shouldn’t be too long or complex. Alternatively, you may be asked to cover your research proposal in more intricate detail or summarise previous research projects. 

At the end of the interview, you will get the opportunity to ask your own questions; prepare some in advance; this will show your interest and enthusiasm, and you will also gain clarification that the program is right for you.

PhD Interviews for Self-Proposed Research Proposals

If you have applied for a position with a self-proposed research proposal, the interview will differ from the interviews for students hoping to complete doctoral research with pre-defined objectives. 

With self-proposed research proposals, you won’t only need to prove your competency, but you will also need to prove the value and originality of your project. Once accepted, you will have freedom over the research you conduct – to a certain extent – however, you will still gain access to similar levels of support, training and resources. 

Even though you have to defend your research proposal, these interviews are typically more flexible and relaxed; you won’t have the pressure of competing for secured funding or affirming why you deserve studentship above other applicants. However, you will face just as much scrutiny as students competing for advertised positions as you will need to confidently convey that the project is viable in terms of time, methodology, and facilities available at the university. 

It is more likely that interviews for students proposing their own research will be more casual; don’t take this as an indication that they are any less important. The postgraduate research panel or your potential supervisor will still need confirmation that you have the right skills and knowledge to go in-depth with research in their field. 

Taking on a new doctoral candidate is a big multi-year commitment for PhD supervisors; you will want to assure them it is worth their time. Typically, this process will include going through points already illustrated in your proposal and expanding on uncovered areas. If funding is on the cards, it will be allocated on a merit basis; with this in mind, elaborate on the value of your project.

How to Prepare for a PhD Interview

Regardless of how your interview is conducted, you will still need to speak about your research proposal and previous work and experience. Spend ample time reviewing your former essays, considering your previous feedback, and going over your proposal with a fine-tooth comb. You should always be prepared to defend any claims you have made with evidence and examples.

During the interview, expect the academic work carried out in your bachelor’s and master’s degrees to be brought up. The merits of them won’t seal your acceptance offer, but discussing your academic background can exhibit your enthusiasm and show how your interest in your research topic developed.

In addition to re-reading your work, take the time to familiarise yourself with the current or recent research carried out by your supervisor. This familiarity will prove that you will relish the opportunity to work with them. If you aren’t sure who your PhD supervisor will be, review the research carried out in the department. 

The lack of originality in research proposals is one of the main reasons for PhD application rejections, which typically happens before the interview stage. However, during your PhD interview, you will also need to verify, if you have self-proposed your own research, that you have investigated the field to ensure your thesis will be completely original.

For PhD projects with pre-set aims, never overlook any of the details of the program. Beyond reviewing the objectives, take an interest in who will be involved, clue yourself in about external funders, and note the available development and training. 

Practice doesn’t only mean perfect; it also means you are committed, capable and confident. Enlist the help of your current academic contacts and peers while practising your interview or presentation material. This is especially important if you don’t have much experience with public speaking or giving presentations.

What To Wear and Bring to a PhD Interview?

Appearance isn’t everything in a PhD interview, but it can go a long way for potential candidates wanting to make the best first impression. Even though academics don’t typically share a similar wardrobe with CEOs during their average workdays, you should present yourself how you would if you were attending a job interview for your dream job. 

The PhD admissions team will inform you if you need to bring anything specific to your interview, such as a presentation. However, if it makes you feel more comfortable and prepared, you can bring hard copies of your previous essays or dissertations, which you can re-read or reference where appropriate. It may also be beneficial to bring a hard copy of your research proposal if you submitted one. 

A notepad and pen will also help you to take notes after you have asked your questions at the end of the interview. Remember, like a job interview, a PhD interview will assess your suitability for the university and the suitability of the university for you. Doctoral research is a big commitment; you will want to ensure that the institution is right for you; your initial interview may be your only chance to explore the campus and meet the staff.

You might also like

phd interview schedule

Do You Get Paid for a PhD?

Do You Get Paid for a PhD? For many students who don’t have the luxury of never worrying about money, one of the main considerations

PhD in Finance

Where Can a PhD in Finance Take Me?

Where Can a PhD in Finance Take Me? In the dynamic world of finance, a PhD is not just an academic accolade; it’s a launchpad

Blonde woman wearing a blue jumper drinking a coffee while deciding to study a PhD in London

Should I Do a PhD in London?

​​Should I Do a PhD in London? Embarking on a PhD journey is a significant decision, one that shapes your academic and professional future. Once

Enquire with us

We are here to help and to make your journey to UWS London as smooth as possible. Please use the relevant button below to enquiry about a course you would like to apply, or to clarify any questions you may have about us and our admission’s process. After you submit your enquiry, one of our advisers will get back to you as soon as possible.

SlideTeam

Researched by Consultants from Top-Tier Management Companies

Banner Image

Powerpoint Templates

Icon Bundle

Kpi Dashboard

Professional

Business Plans

Swot Analysis

Gantt Chart

Business Proposal

Marketing Plan

Project Management

Business Case

Business Model

Cyber Security

Business PPT

Digital Marketing

Digital Transformation

Human Resources

Product Management

Artificial Intelligence

Company Profile

Acknowledgement PPT

PPT Presentation

Reports Brochures

One Page Pitch

Interview PPT

All Categories

Top 10 Ph.D. Interview Presentation Templates With Samples and Examples

Top 10 Ph.D. Interview Presentation Templates With Samples and Examples

Mayuri Gangwal

author-user

Do you know that only 56 % of students complete their Ph.D.? Factors such as students' age, department, and lack of a good mentor sometimes contribute to the non-completion rate. Indeed, the journey to earning a doctoral degree is challenging. It involves years of research and extensive writing. However, most students find a lack of focus and motivation to be the primary reason for their failure. 

If you prepare for the PhD interview, PhD presentation slides can benefit you in several ways. For instance, templates save considerable time and effort and allow to focus on content delivery. Furthermore, for amateurs creating presentations, templates help them to organize content effectively.

At the same time, candidates often face several challenges regarding content creation and delivery. Templates can help streamline the process, but being aware of potential pitfalls is essential. Here are some common challenges associated with using presentation templates for PhD interviews:

In a recent survey, 6 out of 10 students feel nervous and anxious before the final presentation. It is because they find it challenging to communicate the importance of their research effectively. Crafting visually appealing slides can be tricky, especially for those without a design background. Did you find this relatable? Yes, our PhD interview templates can be a valuable solution for you. 

They serve as valuable tools for creating well-structured presentations and assist students in delivering a solid defense for their Doctoral theses. Let's dive in and learn more about these templates and see how they can be valuable resources in your academic journey.

Template 1: Thesis Research Paper Proposal Template

This template can elevate your academic presentations to the next level. It is tailored specially for scholars, researchers, and students. It helps them embark on the rigorous journey of thesis proposal development and ensures that their research proposals are remembered. This template's uniqueness is its visually appealing designs. It integrates text, graphs, and tables and provides a solid structure to your presentation. Whether it's for your academic review or seminars, this template empowers you with confidence and clarity. 

Thesis Research Paper Proposal

Download this template and make your proposal more impactful.

Template 2: Research Proposal Steps Template

Use this template to streamline your research proposal creation process. It is a comprehensive resource covering every crucial aspect of a research proposal. You can use this template to craft an engaging cover letter for your proposal. Thus, this template ensures that your proposal is compelling and professionally presented. Additionally, this template simplifies the process of conveying complex research plans. This template is structured to guide you through the essential steps of the research proposal. It will help you present your research coherently and persuasively. Download now!

Research Proposal Steps

Download this template today and embark on a seamless journey of crafting your thesis proposal.

Template 3: Research Proposal for Thesis Template 

This template can help you effectively present your thesis proposal. It also ensures that you get sponsors for your project by providing a professional look at your proposal. So, this template is a must for someone presenting their hypothesis, as it provides a solid foundation for the presentation. The template encompasses a variety of crucial elements, from the thesis statement to the project stages. 

Research Proposal for Thesis

Download and leverage this template today to focus on critical market components.

Template 4: Abstract for Thesis Research Proposal Template 

This PPT Set helps streamline the complex process of crafting compelling research proposals by providing a structured and intuitive design. The template is divided into two parts. The first consists of six sections briefly describing the thesis. The second part includes a summary and description of the content. Thus, it empowers users to articulate their research objectives and methodologies precisely so that their proposal not only meets but also exceeds expectations.

Abstract for Thesis Research Paper

Download this template today and elevate your academic work to new heights.

Template 5: Research Method Overview Template

This template is designed for ambitious scholars to help them dive into the essence of academic precision. This template helps researchers by providing them with a robust and logical roadmap for their research. This not only increases their efficiency but also helps them select the best research method. This template provides a clear picture of the target audience and how to conduct the study. Thus, this template acts as a catalyst for boosting the proposal's effectiveness. Want to transform your proposal into a compelling narrative that commands attention and respect?

Research Method Overview for Thesis Research Paper Proposal

Download this template today.

Template 6: Method of Data Collection for Thesis Research Paper Proposal

It is the best template for someone looking to elevate their data collection methods. This template provides a clear and professional way to collect data for academic brilliance. It provides a structured framework to articulate the rationale behind the chosen manner. Thus, it is a template and a strategic tool for showcasing your research and methodology. It ensures that your proposal stands out to provide a deep understanding of your work. Additionally, this template helps you communicate complex methodologies in an accessible manner and develop a deeper connection with your audience. 

Method of Data Collection for Thesis Research Paper Proposal

Download this template today and transform your thesis proposal into a masterpiece.

Template 7: Work Plan with Timetable Template 

It is a versatile template that is designed to help professionals across industries. It helps them organize and present their project plans clearly and precisely. The template is divided into three sub-templates to simplify the entire planning phase. The first template includes various activities associated with a specific completion month. It helps you stay organized by outlining different tasks and actions. The second template delves deeper into project activities by outlining a detailed weekly work plan. This way, it provides better visibility and time management. Additionally, it helps you allocate your resources efficiently and prioritize activities. The third and the last templates provide different stages with their names and timeframes, adding to the level of detail and enhancing the proposed research's feasibility.

Work Plan With Timetable- Template 1

Download this template today to take the first step toward achieving project excellence.

Template 8: Implication of Research Template 

This professional template helps you unlock the full potential of your research findings. It is a cornerstone for scholars and professionals eager to convey the significance of their research. The template is Structured as a four-stage process to help students present their research implications. The template's design not only presents data in a captive and visually appealing manner. But it also narrates the story behind your findings and their relevance in real-world applications. Further, this template gives the researcher the chance to explore a variety of angles and helps them consider different aspects of the issue, making research more comprehensive. It further makes research more versatile and applicable to various contexts, which makes it relevant to a broader audience. 

Implication Of Research

Download this template today and bridge the gap between academic research and applications.

Template 9: Aims and Objective of Research Proposal for Thesis Template (Slide 5)

This template is the blueprint for academic success. It is designed to elevate your doctoral thesis proposal. It helps you create a concise and compelling presentation outlining your research objectives. On one side, it highlights the study's objectives, while on the other, it highlights the expected outcome. This way, it ensures that your academic goals are understood easily because lack of clarity may confuse the audience. So, this template sets the stage by explaining what the study aims to achieve. 

Aims and Objective of Research Proposal for Thesis

Download this template today to embark on a journey of research excellence.

Template 10: Dissertation Methodology Template

This comprehensive template can assist students through the complexities of the research approach. It can be their ultimate guide in structuring and presenting their methodology. This template subdivides the entire process into four distinguished subheadings to streamline the process. The first subheading outlines the resources that can be instrumental in research. The second subheading highlights the diversity of the research inputs and helps categorize and organize the gathered data. The following subheading details the analytical techniques for validating your findings. The last, but not least, subheadings discuss the various collection methods and illustrate the strategic approach for gathering comprehensive data. 

Dissertation Methodology

Download this template to set a solid foundation for your dissertation.

Conclusion 

A student takes 4 to 7 years to complete his Ph.D., requiring strategic planning, dedication, and dedication. Additionally, writing and publishing journals is not a cakewalk. It needs exceptional scholars' writing skills along with critical thinking. Our thesis-dissertation templates can open doors to various opportunities and establish you as a credible and competent researcher.

Additionally, our thesis timeline templates help you streamline your project planning. It also bridges the gap between academic reading and research with real-world applications.

Download these templates today and pave the way for a successful and impactful career.

Related posts:

  • How to Design the Perfect Service Launch Presentation [Custom Launch Deck Included]
  • Quarterly Business Review Presentation: All the Essential Slides You Need in Your Deck
  • [Updated 2023] How to Design The Perfect Product Launch Presentation [Best Templates Included]
  • 99% of the Pitches Fail! Find Out What Makes Any Startup a Success

Liked this blog? Please recommend us

phd interview schedule

Must-Have Software Migration Plan Templates with Examples and Samples

Top 10 Hedge Fund Pitch Deck Templates with Samples and Examples

Top 10 Hedge Fund Pitch Deck Templates with Samples and Examples

This form is protected by reCAPTCHA - the Google Privacy Policy and Terms of Service apply.

digital_revolution_powerpoint_presentation_slides_Slide01

Digital revolution powerpoint presentation slides

sales_funnel_results_presentation_layouts_Slide01

Sales funnel results presentation layouts

3d_men_joinning_circular_jigsaw_puzzles_ppt_graphics_icons_Slide01

3d men joinning circular jigsaw puzzles ppt graphics icons

Business Strategic Planning Template For Organizations Powerpoint Presentation Slides

Business Strategic Planning Template For Organizations Powerpoint Presentation Slides

Future plan powerpoint template slide

Future plan powerpoint template slide

project_management_team_powerpoint_presentation_slides_Slide01

Project Management Team Powerpoint Presentation Slides

Brand marketing powerpoint presentation slides

Brand marketing powerpoint presentation slides

Launching a new service powerpoint presentation with slides go to market

Launching a new service powerpoint presentation with slides go to market

agenda_powerpoint_slide_show_Slide01

Agenda powerpoint slide show

Four key metrics donut chart with percentage

Four key metrics donut chart with percentage

Engineering and technology ppt inspiration example introduction continuous process improvement

Engineering and technology ppt inspiration example introduction continuous process improvement

Meet our team representing in circular format

Meet our team representing in circular format

Google Reviews

Interview Invitations: What to Expect

  • First Online: 23 September 2020

Cite this chapter

phd interview schedule

  • Jonathan Sussman 4 ,
  • Jordan Setayesh 5 &
  • Amitej Venapally 6  

1810 Accesses

This chapter is dedicated to the technical and logistical aspects of the MD/PhD interview sessions. These interview sessions can be up to 4 days long and include fancy dinners, medical school tours, talks, and even city tours, in addition to interviews with medical school admissions committee members, MD/PhD admissions committee members, and research faculty. These interview sessions are meant to be fun and exciting. They give applicants a chance to learn about the school and its programs. Therefore, attending these interviews is critical, not only for one’s application but also for the eventual decision that applicants may have to make when selecting which school to attend. By providing advice on planning and scheduling interviews, detailing different events that programs offer, and listing sample interview schedules, this chapter aims to elucidate what applicants should expect during MD/PhD interview visits and to help make these experiences interesting and enjoyable.

This is a preview of subscription content, log in via an institution to check access.

Access this chapter

  • Available as EPUB and PDF
  • Read on any device
  • Instant download
  • Own it forever
  • Compact, lightweight edition
  • Dispatched in 3 to 5 business days
  • Free shipping worldwide - see info

Tax calculation will be finalised at checkout

Purchases are for personal use only

Institutional subscriptions

Grohol MJ. Too much time online: internet addiction or healthy social interactions? Cyberpsychol Behav. 2009;2(5):395–401. https://doi.org/10.1089/cpb.1999.2.395 .

Article   Google Scholar  

Medical School Admissions: Secondary Applications. Kaplan Test Prep. https://www.kaptest.com/study/mcat/medical-school-admissions-secondary-applications/#:~:text=This%20also%20varies%20from%20school,Fee%20Assistance%20Program%20(FAP ). Accessed 11 June 2020.

Download references

Author information

Authors and affiliations.

Perelman School of Medicine, University of Pennsylvania, Philadelphia, PA, USA

Jonathan Sussman

University of Michigan Medical School, University of Michigan, Ann Arbor, MI, USA

Jordan Setayesh

Emory School of Medicine, Emory University, Atlanta, GA, USA

Amitej Venapally

You can also search for this author in PubMed   Google Scholar

Rights and permissions

Reprints and permissions

Copyright information

© 2021 The Editor(s) (if applicable) and The Author(s), under exclusive license to Springer Nature Switzerland AG

About this chapter

Sussman, J., Setayesh, J., Venapally, A. (2021). Interview Invitations: What to Expect. In: The Complete MD/PhD Applicant Guide. Springer, Cham. https://doi.org/10.1007/978-3-030-55625-9_11

Download citation

DOI : https://doi.org/10.1007/978-3-030-55625-9_11

Published : 23 September 2020

Publisher Name : Springer, Cham

Print ISBN : 978-3-030-55624-2

Online ISBN : 978-3-030-55625-9

eBook Packages : Medicine Medicine (R0)

Share this chapter

Anyone you share the following link with will be able to read this content:

Sorry, a shareable link is not currently available for this article.

Provided by the Springer Nature SharedIt content-sharing initiative

  • Publish with us

Policies and ethics

  • Find a journal
  • Track your research
  • for applicants
  • penn admissions info

The tentative schedule for our 2023-2024 interview program is below. Note that  interviews for 2023-2024 are expected to be virtual. Candidates who are selected for an interview will be invited to attend on a particular set of dates, during which their Graduate Group of interest will be highlighted. There is some variation in the schedule, depending on Graduate Group, but the general schedule below is typical.

Most applicants will have 4 or 5 individual interviews: 2 with members of the MD-PhD Admissions Committee and either 2 or 3 with Graduate Group faculty. The number of Graduate Group interviews varies by PhD program.

Plans for additional events include affinity group meet-ups, other social events, and virtual tours. 

All times above are Eastern

**AMCAS has put together a helpful guide geared toward helping applicants prepare for virtual interview that we encourage applicants to review called  Virtual Interviews: Tips for Medical School Applicants .

© The Trustees of the University of Pennsylvania | Site best viewed in a supported browser . | Report Accessibility Issues and Get Help | Privacy Policy | Site Design: PMACS Web Team. | Sitemap

ThePhDHub

How to Prepare a PhD Research Plan/Schedule?

PhD research plan is a structured schedule for completing different objectives and milestones during a given timeframe. Scholars are usually unaware of it. Let us find out how to prepare it. 

Between March 2021 to 2022, I read almost 15 different research proposals from students (for their projects) and only a single one, I found, with a comprehensive research plan for 3 years. Which is still not, kind of practical, probably copied from other students. 

Such entities are not known to over 90% of students, if some know that because their university asked for but unfortunately, this basic procedure lacks penetration among students. I don’t know the exact reason, but students lack a basic understanding of the research process. 

Meaning, that they don’t know or perhaps don’t complete their course work needly. PhD research requires many documents, SOPs and write-ups, before even starting it. For example, a rough research plan, research proposal, initial interview, competence screening, grant proposal and so on. 

However, the requirement varies among universities and thus knowledge regarding basic procedures often also varies among students. So I’m not blaming students but certainly, it is the fault of the university side, as well.  

When you come up with a research proposal with a research schedule or entire plant, certainly it will create a positive image and good reputation. So it is important. But how to prepare it? 

Hey, there I’m Dr Tushar, a PhD tutor and coach. In this article, we will understand how we can prepare a structured plan for the PhD research and how to execute it. 

So let’s get started.  

How to prepare a PhD research plan/schedule?

A PhD research plan or schedule can be prepared using the GANTT chart which includes a month, semester or year-wise planning of the entire PhD research work. 

First, enlist goals and objectives.

It’s not about your research objective enlisted in your proposal. I’m talking about the objectives of your PhD. Take a look at some of the objectives.

Note that these are all the objectives that should be completed during the PhD, but not limited to a specific subject. Note you have to show how you can complete or achieve each objective during the entire tenure of your work. 

And that is what the plan/schedule is all about. Next, explain the time duration. The time required to complete each goal, roughly. For example, a semester or a year to complete the course work or 4 to 8 months for completion of ethical approval. 

Now two things must be known to you, at this point in time. 

  • First, enlist the time required to complete each objective, as aforementioned. 
  • Second, what goals would you complete during each semester?

For instance, course work takes a semester to complete, but during the period a scholar can also craft their PhD research title, research proposal, ethical approval and grant proposals. 

Now it is also crucial to know that there is no time bound to complete goals, but it should be completed as you explained. Let’s say you can plant it for 3 years, 4 or even 5 years depending on the weightage of your work. 

In summary, the answer to the question of how to prepare a research plan is, 

  • Enlist your goals or objectives. 
  • Decide the time required to complete each goal.
  • Prepare a GANTT chart.  

Now you have prepared zero-date planning for your research but how to present it? The answer is a GANTT chart.   

GANTT chart for PhD research plan: 

GANTT chart is a task manager and graphical presentation of how and how many tasks are completed or should be completed against a given time duration. Take a look at the image below. 

The example of the GANTT chart.

How can you prepare one?

Open MS Excel (on Windows) or numbers (on Mac).

Enlist goals or objectives in a column. 

Enlist years (duration of PhD) in a row and bifurcate them into individual semesters. You can also prepare a month-wise plan, that’s totally up to you. In my opinion, semester-wise planning is good because research is a lengthy and time-consuming process. So monthly planning would not work. 

To make a chart more attractive and readable use colors, as I used. Now mark a ‘cell’ against a column and row showing the objective which you are going to complete in a semester. Take a look. 

After the end of this, your GANTT chart would look like this. 

A screenshot of an ideal GANTT chart.

You can prepare a month-wise planning, individual semester-wise planning and goal-wise planning etc. I will explain these things in upcoming articles on 5 different types of GANTT charts for PhD.  

Custom writing services: 

If you find difficulties in preparing a research plan, synopsis, proposal or GANTT chart. We can work on behalf of you. Our costume services are, 

  • Synopsis writing 
  • Project writing 
  • Research proposal writing 
  • Research planning and GANTT chart preparation. 

You can contact us at [email protected] or [email protected] to get more information. 

Wrapping up: 

Planning and executing a research schedule are two different things. Oftentimes, students just prepare as per the requirements and then do work as per their convenience. Then they are stuck in one place and just work around the time. 

Plan things. Make your own GANTT chart, put it on your work table or stick it on a wall so that you can see it daily. Try to achieve each goal in time. Trust me things will work and you will complete your PhD before anyone else.  

Dr Tushar Chauhan

Dr. Tushar Chauhan is a Scientist, Blogger and Scientific-writer. He has completed PhD in Genetics. Dr. Chauhan is a PhD coach and tutor.

Share this:

phd interview schedule

  • Share on Facebook
  • Share on Twitter
  • Share on Pinterest
  • Share on Linkedin
  • Share via Email

About The Author

' src=

Dr Tushar Chauhan

Related posts.

Why is it called a Doctor of Philosophy?

Why is it called a Doctor of Philosophy?

PhD viva voce session

Preparing for a PhD Viva

Leave a comment cancel reply.

Your email address will not be published. Required fields are marked *

Save my name, email, and website in this browser for the next time I comment.

Notify me of follow-up comments by email.

Notify me of new posts by email.

  • Payment Link
  • Admissions Blog
  • Faculty Members
  • Non Teaching Staff
  • Faculty Login
  • Admission Portal
  • Apply For SAU Internet login
  • Apply For SAU Email ID Form
  • Room Booking
  • SAU Documents
  • Password Change / Reset

South Asian University

A university established by saarc nations.

  • Key University Functionaries
  • University Administration
  • Past Presidents
  • Admission Notice 2024
  • Admission Procedure
  • PhD Programmes
  • General Eligibility
  • Master Programmes
  • Application Fee, Dates and Test Centres
  • Tuition and Other Fees
  • Student Visa
  • Hostel and Medical Facilities
  • Scholarships & Financial Support
  • Admission FAQs
  • Through Entrance Mode
  • Through Direct Mode
  • Department of Computer Science
  • Department of Mathematics
  • Faculty of Economics
  • Faculty of Legal Studies
  • Faculty of Life Sciences & Biotechnology
  • Department of International Relations
  • Department of Sociology
  • Institute of South Asian Studies
  • Centers & Labs
  • Academic Calendar
  • PhD Notification
  • Alumni Association
  • South Asia Introduction
  • Visa FRRO Registration
  • Proctorial Committee
  • Grievance Redressal
  • Support Contacts
  • Anti-Sexual Harassment Policy
  • Key Documents
  • SAU Connect
  • Press Releases
  • Media Downloads
  • Media Coverage
  • Annual Reports
  • Second Convocation-2017
  • First Convocation – 2016
  • Faculty of Mathematics & Computer Science
  • Administrative
  • Admissions and Examinations
  • Engineering
  • Estate & Procurement
  • Faculty Assistant
  • Health Center
  • Information & Communication Technology (ICT)
  • Research & Development
  • Job Announcements
  • Research Positions

phd interview schedule

PhD Interviews Schedule 2024

The PhD interviews for shortlisted candidates (through Entrance Mode as well as Direct Mode) will be held as per the following schedule. For Indian candidates, the interviews will be held at the SAU Campus while for non-Indian Candidates, these will take place online. Further details will be communicated in due time:

Doctor Elena GR

The essential guide to developing a boomtastic semi-structured interview schedule for your Interpretative Phenomenological Analysis (IPA) research study

Image of question marks

Written by Elena Gil-Rodriguez

Data collection for your interpretative phenomenological analysis (ipa), aka: how do i come up with some awesome interview questions.

Updated February 2022

Semi-Structured Interviews are the prince (or princess) of data collection methods for your IPA

I am well aware that many of you will be conducting semi-structured interviews (SSIs) as your data collection method for your Interpretative Phenomenological Analysis (IPA) research study. After all, it is the exemplar and the most common data collection method for this type of study.

As I have already covered developing your IPA research question (RQ) in a previous article, I will now turn my attention to constructing an effective interview schedule for your data collection.

A high-quality interview schedule (or interview guide) will give you the best chance of gathering high-quality data for your IPA

Another no-brainer but the quality of your preparation for interviewing process will really help ensure that you do not get caught in a ‘garbage in, garbage out’ situation with your data.

Firstly, make sure you have mapped out your topic area sufficiently with the extant literature and have therefore done your background research, and are clear on the gaps that you are looking to fill.

The reason for this is that it will inform what you ask in the interview and will help you avoid going over the same ground that has already been covered before in previous research.

Spend time on developing your interview schedule!

It is well worth investing the time and energy in optimising your schedule as far as is humanly possible to avoid the ‘garbage in and garbage out’ dilemma.

Practical tips for constructing your IPA interview schedule

Identify the main themes relevant to your RQ and start to develop specific questions around each theme.

Start freewriting, attempting to construct as many open-ended questions as you can.

You will eventually whittle this down to a shorter set of approximately ten questions – remember you will have lots of space for prompts and probes (see my next article on interviewing) to extend the conversation further.

So, while only having ten questions might feel anxiety-provoking, I can assure you it is more than enough if you mine the interview as much as possible by skilfully employing your prompts and probes to keep the conversation going and achieve greater depth from your participant. See this article to help with this aspect of your interview technique.

Once you have constructed your short-list of questions, check them over and ask yourself:

  • Are they suitable for the topic area (e.g., are they relevant as per above)?
  • Will my interview questions will help me answer my RQ?
  • Is my first question broader and ‘scene setting’ to help settle my participant into the interview and develop rapport?
  • Are my questions suitable for my participant group (e.g., are they worded correctly for specific groups such as young people, children, vulnerable participants etc)?
  • Are they understandable (i.e., not confusing to the participant)? Do try them out on your geek-love study buddy to check comprehension and that you haven’t asked two questions in one (this is a common and very human habit)
  • Can I get some broader peer feedback and take my schedule to my local IPA regional group?
  • Can I organise a ‘reverse interview’ and try these questions on myself? This is where you get a buddy to interview you using your schedule so that you have some idea of how it might be to be the participant. NOTE: Doing a reverse interview can also help to reveal some of your fore conceptions as part of the hermeneutic circle and is sometimes referred to as a ‘bracketing interview’  

What if the areas I am interested in are not covered by the participant in the interview?

Don’t fret!

IPA SSIs are participant-led and we are looking for their take on the phenomenon , not our take on the phenomenon.

We therefore need to allow the participant to set the parameters of the topic area and describe their experience, not what you assume their experience is .

You never know, you might uncover something quite novel and exciting that you had not anticipated by giving them the space to lead the interview! 

You can, of course, introduce your topic of interest later in the interview if the participant does not raise it themselves. In this way, you will have let them set the parameters of that experience and yet can also probe into your area of concern once they have done this on their own terms.

What resources can I access to help me develop my IPA interview schedule?

Of course, the Smith, Flowers, and Larkin (2009) ‘bible’ pages 59 to 62 gives a great overview of types of questions with a bucketload of examples. I encourage you to access this helpful resource.

S mith, J.A., Larkin, M., & Flowers, P. (2009) . Interpretative phenomenological analysis: Theory, method and research . London: Sage

In the 2022 Second Edition , Chapter 4, ‘Collecting data’ is where you want to immediately head for guidance.

Braun and Clarke also have an excellent chapter on interviewing, including constructing a guide (schedule) in their wonderful 2013 text. This is also highly recommended.

Braun, V., & Clarke, V. (2013). Successful qualitative research: a practical guide for beginners . London: Sage .

Other references that may be useful to you include:

Brinkmann, S., & Kvale, S. (2018). Doing interviews . (2nd ed.) London: Sage

Warren, C.A.B. (2011). Qualitative interviewing. In J.F. Gubrium & J.A. Holstein (Eds.), Handbook of Interview Research (pp.83-102). London: Sage

The need to systematically develop your interviewing skills

Finally, just to mention that I obviously go into this area in a great deal of detail on my Supercharge Your Semi-Structured Interviewing workshop . I lead you by the hand through the process of constructing a high-quality interview schedule and preparing extensively for the interview process. We also spend a significant amount of time practicing interview skills in a group exercise.

I encourage you to attend one of my workshops dedicated specifically to helping you achieve the required skillset to conduct high-quality IPA interviews and therefore optimise the data that you gather for your study. You cannot make a silk purse from a sow’s ear and good quality data that achieves sufficient experiential depth is essential to make your life easier at the point of conducting your analysis.

As Professor Smith repeats, time and time again: ‘Interviewing is a critical part of the process and it can require considerable time to develop expertise’ (Smith, 2011, p. 23)

Reference: Smith, J.A. (2011). Evaluating the contribution of interpretative phenomenological analysis. Health Psychology Review, 5 , 9-27.

This is an area that students typically struggle with, just as I did! This is why I developed my Supercharge Your Semi-Structured Interviewing workshop – to help you develop your ability in this vital area. Developing these skills and grappling with this task in your research journey are all part of the process of becoming a qualitative researcher. I encourage you to see it as such rather than a monkey on your back that is whispering doom and gloom into your ear!

Until next time!

To your research success, Elena

Copyright © 2020-24 Dr Elena Gil-Rodriguez

T hese works are protected by copyright laws and treaties around the world. Dr Elena GR grants to you a worldwide, non-exclusive, royalty-free, revocable licence to view these works, to copy and store these works and to print pages of these works for your own personal and non-commercial use. You may not reproduce in any format any part of the works without my prior written consent. Distributing this material in any form without permission violates my rights – please respect them.

The information contained in this article or any other content on this website is provided for information and guidance purposes only and is based on Dr Elena GR’s experience in teaching, conducting, and supervising IPA research projects. All such content is intended for information and guidance purposes only and is not meant to replace or supersede your supervisory advice/guidance or institutional and programme requirements, and are not intended to be the sole source of information or guidance upon which you rely for your research study. You must obtain supervisory and institutional advice before taking, or refraining from, any action on the basis of my guidance and/or content and materials. Dr Gil-Rodriguez disclaims all liability and responsibility arising from any reliance placed upon any of the contents of my website or associated content/materials. Finally, please note that the use of my content/materials does not guarantee any particular grade for your work.

You may also like…

Qualitative data collection in the time of coronavirus…

Qualitative data collection in the time of coronavirus…

Updated February 2022 I had plenty of time in 2020, and indeed much of 2021, to muse on conducting qualitative...

The complete guide to online interviewing for your Interpretative Phenomenological Analysis (IPA) or other qualitative research study: episode one

The complete guide to online interviewing for your Interpretative Phenomenological Analysis (IPA) or other qualitative research study: episode one

Updated February 2022 I know that I have already written a brief article about my musings on data collection in the...

Top tips and tricks for tackling online interviewing for your Interpretative Phenomenological Analysis (IPA): episode two

Top tips and tricks for tackling online interviewing for your Interpretative Phenomenological Analysis (IPA): episode two

Updated February 2022 After the last mammoth VC article, I am returning as promised with Episode Two. This time, I aim...

Up-level your semi-structured interview technique for your Interpretative Phenomenological Analysis (IPA): using prompts and probes to keep the conversation going and achieve greater depth

Up-level your semi-structured interview technique for your Interpretative Phenomenological Analysis (IPA): using prompts and probes to keep the conversation going and achieve greater depth

Updated February 2022 Following on from my article about developing a boomtastic interview schedule for your...

Privacy Overview

IGNOUHelp.in

A Leading Portal for IGNOU Students

IGNOU PhD Interview Schedule 2024 (OUT): Check Date and Timing

IGNOU PhD Interview Schedule 2024 – IGNOU University holds interviews every year for the selection of candidates for Ph.D. (Doctor of Philosophy) Programmes. Candidates may take admission to any of the IGNOU PhD Courses offered by the University. To get Admission to PhD, candidates have to appear in the entrance exam conducted by IGNOU and then they will be selected through an Interview which will be held at the main campus of the university.

If candidates have an M.Phil degree or cleared JEST of DAE or UGC-NET or GATE of IIT or if they have a minimum 5-year working experience in Teaching/Practical/Industry/Professional then those candidates will be exempted from appearing in the entrance exam and they will directly call for an interview. If you haven’t any experience then you have to pass an entrance exam and then you will call for an interview for selection.

  • IGNOU PhD 2024 – Admission, Result
  • IGNOU Fee Structure 2024
  • IGNOU PhD Prospectus 2024

Required Documents for IGNOU PhD Interview 2024

IGNOU PhD Interview

  • SSC certificate for date of birth.
  • Graduation and Post Graduation Mark sheets and Certificates.
  • Exemption proof in case of A1 category applicants with the validity period of NET/JRF/GATE.
  • Caste certificate, if required.
  • PH Certificate, if required.
  • Course Completion Certificate along with Mark sheets (55% marks in course work is compulsory) in case of students who have already completed MPhil.

IGNOU Phd Interview Dates 2024

IGNOU has already conducted the IGNOU PhD Entrance Examination for July 2023 and the result for the same has also been declared by the university online. Furthermore, the university has started to release the IGNOU PhD Interview list for various disciplines from 13th March 2024. The students from respective disciplines should check the PhD Interview list regularly to know their interview dates.

IGNOU PhD Admission Interview Schedule 2024

The university has officially started to release the schedule for the IGNOU PhD Interview which will be finalized in the selection process to shortlist the candidates for admission to the concerned PhD program. The candidates have to find their interview schedule below to attend the same on the given schedule and at the given venue.

You can keep connected to check other category interview Schedules and Candidate List to reach IGNOU Campus to attend your interview with the exact date.

Related Posts:

  • IGNOU PhD Admission 2024: Interview Result (OUT),…
  • IGNOU Placement 2024 | Eligibility, Salary,…
  • IGNOU Exam Time Table June 2024 (OUT), IGNOU Time…
  • IGNOU Classes 2024 Start Date, Schedule & Timing
  • IGNOU 37th Convocation 2024: Registration, Date & Venue
  • IGNOU Hall Ticket June 2024, IGNOU June 2024 Hall Ticket

22 thoughts on “IGNOU PhD Interview Schedule 2024 (OUT): Check Date and Timing”

when will be interview schedule for PhD tourism and hospitality service management of qualified entrance exam candidate held on 7th January 2024?

I attended interview on 12Jun 23,for phd MGT, what is tentative date for result.

When will be come lifescience phd interview result

Phd Commerce EWS category interview schedule not found. WHEN WILL IT BE CONDUCTED?

What about management PhD interview date

When tourism interview list and date will be declare

When will the interview for Ph.D. in Sanskrit take place?

I qualified phd entrance exam in hindi subject . I want check schedule and selected candidates ki list

i have passed PHD ENTRANCE EXAM OF EDUCATION..when will BE THE interview DATE

When and where IGNOU PHD entrance exam of English Literature as a subject of its interview will be commenced?

will I be informed in my mail id , on the interview call in PHDSOC after NTA entrance test on 4th October

When the Ph.D interview result will be declared ? atleast kindly intimate the expected date. eagerly waiting for the result

When the result of PhD interview in management will be declared???

when the Ph.d result will be declared? waiting eagerly .

P.hd environmental science ka interview kab hoga Koi information dijiye

When will the result of PhD interview of management declared?? Kindly let me know

When will the result of Ph.d Interview of Library and Information Science declared. Let me know the result for it.

i gave interview for phd in environmental sciences. kindly inform me about result.

I also qualify p.hd entrance in environmental science kindly inform me about interview date

i have qualified the entrance exam for Phd Environemntal sciences. please tell me when will the interview date will come and is the university send a mail or any messege to the qualifying candidates regarding the interview

Aapko interview date ka pata chala

http://ignou.ac.in/ignou/aboutignou/icc/ru/phdinterview Visit IGNOU official site..latest news…PhD interview schedule 2021….click on environment science… download pdf file of your interview schedule. Click on word “Environmental science”. Then, download your interview date 20-22 April ,2022

Leave a Reply Cancel reply

Your email address will not be published. Required fields are marked *

IMAGES

  1. FREE 10+ Research Interview Schedule Templates in PDF

    phd interview schedule

  2. FREE 10+ Research Interview Schedule Templates in PDF

    phd interview schedule

  3. FREE 10+ Research Interview Schedule Templates in PDF

    phd interview schedule

  4. FREE 10+ Research Interview Schedule Templates in PDF

    phd interview schedule

  5. Common PhD Interview Questions

    phd interview schedule

  6. Easily Track Research & Evaluation Interviews in Excel

    phd interview schedule

VIDEO

  1. PhD Admission 2024 || Latest Update

  2. Lu phd interview schedule out#phd #lucknow #study #ugcnetjrf #lu

  3. How to prepare for PhD interviews||what to study||Books to study||IITs&IISc Interviews||

  4. PhD Interview/Viva Schedule of Lucknow University #lucknowuniversity #phd #shorts

  5. PhD

  6. PhD Scholarships Interview Questions and Answers

COMMENTS

  1. To ace your Ph.D. program interviews, prepare to answer—and ...

    To ace your Ph.D. program interviews, prepare to answer—and ask—these key questions. You've made it to the last step of the Ph.D. application process: the interview. Congratulations! But amid the excitement and butterflies, don't neglect the crucial next step: preparation. Grad school interviews—in which aspiring graduate students meet ...

  2. PhD Interview Questions and Answers

    Be honest about the things you find challenging, but identify them as training needs and discuss how you expect to improve upon them as part of your PhD. Do answer: I feel that I'm a good written communicator. My existing academic and professional work demonstrates an ability to put forward ideas clearly and concisely.

  3. The PhD Interview

    Depending on the format for your PhD interview it could involve: A formal question and answer session in front of a postgraduate recruitment panel. A presentation, based on your research proposal or area of expertise. A one-to-one discussion with your prospective supervisor.

  4. PhD Interview Questions and Answers (13 Questions + Answers)

    Most PhD applications include an interview. This allows your university (and perhaps even your prospective supervisor) to discuss the PhD with you in more detail. This article lists some of the most common PhD interview questions along with their answers. The goal is to help you prepare for a PhD interview and pass with flying colors.

  5. Preparing For Your PhD Interview

    A PhD interviewer commonly asks for the interviewee to create a short presentation (3-5 slides) to bring the supervisors up to date with what relevant experience you have, to learn a bit about you as a person and to see your motivation for taking their project. It usually only takes up the first few minutes of the interview.

  6. PhD Interview Weekend: A guide for aspiring scientists

    General PhD Interview Weekend Itinerary. Day 1 (Thursday or Friday) - usually a day to meet people and get comfortable. 12:00pm Arrive somewhere on campus or at hotel and get settled in. 1:00-6:00pm Orientation, seminars and poster session. 6:00-8:00pm Dinner with faculty and/or graduate students, with beer and wine.

  7. Common PhD Interview Questions

    Common PhD Interview Questions. In this guide, we'll share 11 common PhD interview questions and our suggestions on how to answer them. A PhD interview is an essential step in securing a doctorate position. This is because it enables the prospective supervisor to get to know you better and determine whether you'd be a good fit for the project.

  8. Top 15 PhD Interview Questions and Answers

    Research Experience and Background Interview Questions. 1. Tell me about your research experience and background in your field. How to Answer: Start with a brief overview of your academic and research journey, highlighting key milestones, projects, and publications. Emphasize your expertise, the relevance of your work to the PhD program, and ...

  9. "Tell Me About Yourself" PhD Interview Question & Expert Answers

    The "tell me about yourself" PhD interview question is among the most common and difficult graduate school interview questions, but the right prep will help you start the interview and set the tone for the ... Schedule a free strategy call now. Booked times today. Trusted by students. Expert Advice, Delivered! Get admissions tips and advice ...

  10. Questions to Ask During Your PhD Interview

    You're asking this to firstly work out how experienced the professor is at supervising students, based purely on the numbers previously supervised. The reason to ask the second question of how many students gained PhDs is to get an idea of the supervisor's track record of successful supervision. The lower the percentage of students that ...

  11. Interviewing and Talking with Prospective Faculty

    2. During the interview. For campus visits, dress appropriately (usually business casual, but be comfortable); be on time; organize your papers (e.g., résumés/CVs, slides or images, questions). Even for phone/Skype interviews, dressing professionally will help you to mentally prepare for the interview.

  12. How To Prepare for a PhD Interview

    September 26, 2023. Home » PhD » How To Prepare for a PhD Interview. So, you've written your personal statement, you've sent through your application and now you've been invited to a PhD interview, the first step is to congratulate yourself; only 30 - 50% of PhD applicants make it through to the initial interview stage.

  13. Top 10 Ph.D. Interview Presentation Templates With Samples ...

    Template 7: Work Plan with Timetable Template. It is a versatile template that is designed to help professionals across industries. It helps them organize and present their project plans clearly and precisely. The template is divided into three sub-templates to simplify the entire planning phase.

  14. Apply

    Application Contacts. Application questions: Please refer to the Harvard Griffin GSAS Admissions website, call 617-496-6100 (please call between 2:00 p.m. and 5:00 p.m. Eastern Time, Monday through Friday), or contact [email protected] . Degree program questions: If you have questions about the BBS Program, please reach out to Danny ...

  15. 21-Point Interview Presentation Checklist For PhDs

    Always ask for clarification. And, once you know what they want, it's time to prepare. Here is a 21-point checklist for PhDs preparing for an industry interview presentation…. 1. Know your audience and tailor your talk to them.

  16. Interview Invitations: What to Expect

    Although the interview process is fun, we would like to warn you that it can be exhausting, especially if you schedule back-to-back interviews and are continually traveling across the country. Unlike the MD-only interview process, which usually just involves half a day, the MD/PhD interview sessions can last 2-3 days, depending on the school.

  17. PhD Interview tips? : r/gradadmissions

    I would suggest being specific about your research or professional experience (e.g., your particular roles in the past projects). If you worked as a team, be prepared to be asked about your collaboration experience. Also, you should be able to speak about your research interests (be specific, e.g., talk about the scenarios you want to work on).

  18. interviews

    The tentative schedule for our 2023-2024 interview program is below. Note that interviews for 2023-2024 are expected to be virtual. ... The number of Graduate Group interviews varies by PhD program. Thursday . 11:00 to 12:15: Orientation for the MD-PhD Program and Medical School: 12:15 to 1:15: Break:

  19. PDF Appendix 1: Semi-structured interview guide

    The interview will take place in person at a specific location or over the phone. Participants in the UK have the option of an in-person or teleconference interview, and all other participants will have teleconference ... After the study has finished, the results will be written up as part of the PhD research thesis of Linda

  20. How to Prepare a PhD Research Plan/Schedule?

    A PhD research plan or schedule can be prepared using the GANTT chart which includes a month, semester or year-wise planning of the entire PhD research work. First, enlist goals and objectives. It's not about your research objective enlisted in your proposal. I'm talking about the objectives of your PhD.

  21. PhD Interviews Schedule 2024

    PhD Interviews Schedule 2024. The PhD interviews for shortlisted candidates (through Entrance Mode as well as Direct Mode) will be held as per the following schedule. For Indian candidates, the interviews will be held at the SAU Campus while for non-Indian Candidates, these will take place online. Further details will be communicated in due time:

  22. The essential guide to developing your IPA study interview schedule

    Practical tips for constructing your IPA interview schedule. Identify the main themes relevant to your RQ and start to develop specific questions around each theme. Start freewriting, attempting to construct as many open-ended questions as you can. You will eventually whittle this down to a shorter set of approximately ten questions ...

  23. IGNOU PhD Interview Schedule 2024 (OUT): Check Date and Timing

    IGNOU PhD Interview Schedule 2024 - IGNOU University holds interviews every year for the selection of candidates for Ph.D. (Doctor of Philosophy) Programmes.Candidates may take admission to any of the IGNOU PhD Courses offered by the University. To get Admission to PhD, candidates have to appear in the entrance exam conducted by IGNOU and then they will be selected through an Interview which ...